Surgery 1

July 19, 2017 | Author: Ong Kie | Category: Shock (Circulatory), Breast Cancer, Stomach, Pancreatic Cancer, Thyroid
Share Embed Donate


Short Description

surgery review questions...

Description

SURGERY 1.

A man sustained severe blunt injuries in a car crash twelve hours ago. The activity of this hormone is now expected to be decreased in this patient: A. insulin B. cortisol C. epinephrine D. aldosterone (Classification - Application; Source – Schwartz‟s Textbook of Surgery (8th edition): pp. 25-29) 2.

A young woman is debilitated by pruritus and burning sensation from her presternal keloid. The recommended initial treatment is: A. topical application of silicone sheets B. intralesional corticosteroid injection C. surgical excision D. low-dose radiation (Classification - Recall; Source – Schwartz‟s Textbook of Surgery (8th edition): pp. 241-242) 3.

A patient is placed on NPO in preparation for an elective major abdominal surgery. The body‟s preferred initial fuel source during the fasting state is: A. hepatic glycogen B. skeletal muscle glycogen C. muscle protein D. body fat (Classification - Recall; Source – Schwartz‟s Textbook of Surgery (8th edition): p. 24) 4.

A 48-year-old woman has prolonged ileus after surgery for an obstructed duodenal ulcer. The problem is probably due to: A. hypochloremia B. hypocalcemia C. hypomagnesemia D. hypokalemia (Classification - Application; Source – Schwartz‟s Textbook of Surgery (8th edition): pp. 48-49) 5.

A man with chronic prepyloric ulcer appears weak after repeated bouts of non-bilious vomiting over the past 3 days. Fluid therapy should be started using: A. Normosol M B. Lactated Ringer‟s solution C. Normal saline solution D. Hypertonic saline solution (Classification - Application; Source – Schwartz‟s Textbook of Surgery (8th edition): pp. 51-52) 6.

A multiply injured patient has persistently low urine output. The oliguria is most likely due to prerenal failure rather than acute tubular necrosis if the tests reveal: A. low urine specific gravity B. low urinary excretion of sodium C. low BUN/creatinine ratio D. low creatinine clearance (Classification - Application; Source – Schwartz‟s Textbook of Surgery (8th edition): pp. 348-349) 7.

A woman admitted to the ICU for severe acute pancreatitis begins to manifest paresthesia of the face and extremities, muscle cramps, and a positive Chvostek‟s sign. These are probably due to: A. hypocalcemia B. hypokalemia C. hypomagnesemia D. hypophosphatemia (Classification - Application; Source – Schwartz‟s Textbook of Surgery (8th edition): p. 50) 8.

Increased nutritional support appears to aggravate tachypnea in a septic patient with respiratory failure. Lessening the amount of this substrate may alleviate the problem: A. carbohydrate B. fat emulsion C. essential amino acids D. non-essential amino acids (Classification - Recall; Source – Schwartz‟s Textbook of Surgery (8th edition): pp. 28-29) 9.

Administration of this amino acid is expected to be beneficial to a patient suffering from damage to intestinal mucosa due to adjuvant chemotherapy for esophageal cancer: A. arginine B. valine C. leucine D. glutamine (Classification - Recall; Source – Schwartz‟s Textbook of Surgery (8th edition): p. 38) 10. Early supplementation of this vitamin is recommended to promote wound repair in a patient with extensive seconddegree flame burns: A. A B. C C. D D. E (Classification - Application; Source – Schwartz‟s Textbook of Surgery (8th edition): p. 210)

11.

A diabetic 41-year-old man is admitted for necrotizing fasciitis in the perineum. A high dose of this antibiotic is generally included in the initial antimicrobial therapy because of concern for clostridial pathogens: A. metronidazole B. vancomycin C. penicillin G D. aminoglycoside (Classification - Recall; Source – Schwartz‟s Textbook of Surgery (8th edition): p. 122) 12.

Transfusion of properly cross matched blood is begun on a man admitted for massive bleeding from erosive gastritis and thirty minutes later, he develops urticaria and fever. This should be administered to the patient: A. antihistamine B. mannitol C. furosemide D. sodium bicarbonate (Classification - Application; Source – Schwartz‟s Textbook of Surgery (8th edition): pp. 79-80) 13.

A patient under anticoagulation therapy using warfarin, who is set to undergo surgery for acute cholecystitis, has decreased prothrombin concentration. Warfarin can be reversed by parenteral dose of: A. protamine sulfate B. vitamin K C. EACA D. hydroxyurea (Classification - Recall; Source – Schwartz‟s Textbook of Surgery (8th edition): pp. 73-74) 14.

Arterial blood gas analysis is performed on a patient just admitted with a diagnosis of severe acute pancreatiti s. This reveals a pH of 7.30 and low levels of bicarbonate and pCO 2. The most urgent part of management is: A. volume resuscitation B. intravenous bicarbonate C. calcium infusion D. mechanical ventilation (Classification - Application; Source – Schwartz‟s Textbook of Surgery (8th edition): pp. 50-51) 15.

Seven days after surgery for a perforated appendicitis, the primarily closed incision is noted to be erythematous, slightly swollen and tender. The appropriate treatment is: A. local heat therapy B. topical antibiotics C. new systemic antibiotics D. incision and drainage (Classification - Application; Source – Schwartz‟s Textbook of Surgery (8th edition): pp. 119-120) 16.

A man is brought to the E.R. with blood spurting from a hacking wound in the in the distal right thigh. He is alert and has a systolic BP of 100 mmHg. What is the initial management step? A. apply direct pressure on the wound with sterile gauze B. apply digital pressure on proximal femoral artery C. apply a thigh tourniquet above the wound D. open the wound and clamp the bleeders (Classification - Application; Source – Schwartz‟s Textbook of Surgery (8th edition): p. 74) 17.

A young man presents to the E.R. with a stab wound in the left chest. Examination reveals subcutaneous emphysema and absent breath sounds on the left chest; the trachea is shifted to the right. What is the probable diagnosis? A. massive hemothorax B. tension pneumothorax C. cardiac tamponade D. flail chest (Classification - Application; Source – Schwartz‟s Textbook of Surgery (8th edition): p. 131) 18.

A 58-year-old woman on NSAID therapy for arthritis has a 6-hour history of epigastric pain that has progressively become severe. Her abdomen is distended, rigid and diffusely tender. This diagnostic test should be done first: A. upright chest x-ray B. esophagogastroduodenoscopy C. abdominal ultrasound D. abdominal CT scan (Classification - Application; Source – Schwartz‟s Textbook of Surgery (8th edition): pp. 959-960) 19.

After aggressive fluid therapy, a trauma victim shows a systolic BP of 110 mm Hg, cold extremities, rapid but strong peripheral pulse, and a central venous pressure of 12 cmH 2O. The patient has: A. increased systemic vascular resistance B. decreased stroke volume index C. decreased cardiac index D. excessive cardiac preload (Classification - Application; Source – Schwartz‟s Textbook of Surgery (8th edition): p. 132) 20.

An elderly man admitted with a diagnosis of sigmoid volvulus has a markedly distended, non-tender abdomen and hyperactive bowel sounds. The initial treatment is: A. endoscopic detorsion B. detorsion via laparotomy C. decompressing transverse loop colostomy D. Re section of involved segment of colon (Classification - Recall; Source – Schwartz‟s Textbook of Surgery (8th edition): p. 1098)

21.

Pancreatic necrosis is suspected in a 49-year-old woman who has not improved despite 3 days of intensive care for acute pancreatitis. The diagnosis is best established through: A. C-reactive protein measurement B. abdominal ultrasound C. contrast-enhanced CT scan D. CT-guided percutaneous biopsy (Classification - Recall; Source – Schwartz‟s Textbook of Surgery (8th edition): pp. 1234-1238) 22.

A chronically constipated 67-year-old woman presents with acute left lower quadrant (LLQ) abdominal pain, LLQ direct and rebound tenderness, and fever. The appropriate diagnostic examination is: A. transvaginal ultrasound B. abdominal CT scan C. proctosigmoidoscopy D. barium enema (Classification - Application; Source – Schwartz‟s Textbook of Surgery (8th edition): pp. 1082-1083) 23.

After an elective hemicolectomy for colon cancer, a 78-year-old man remains hypotensive and tachycardic. He has distended neck veins, cold skin, oliguria, and elevated central venous pressure. He apparently is suffering from this type of shock: A. hypovolemic B. cardiogenic C. septic D. neurogenic (Classification - Application; Source – Schwartz‟s Textbook of Surgery (8th edition): pp. 95-100) 24.

For the past 6 weeks, a 67-year-old man has been asymptomatic except for constipation after a course of antibiotic therapy for left lower quadrant abdominal pain. He should undergo: A. CEA determination B. barium enema C. colonoscopy D. abdominal CT scan (Classification - Application; Source – Schwartz‟s Textbook of Surgery (8th edition): pp. 1082-1083) 25.

A 32-year-old-man with a week‟s history of fever presently has right lower quadrant abdominal pain and tenderness and bloody diarrhea. The most probable diagnosis is: A. typhoid ileitis B. amebic colitis C. ileocecal TB D. intussusception (Classification - Application; Source – Schwartz‟s Textbook of Surgery (8th edition): pp. 1049-1050) 26.

A 75 year old woman in the ICU after undergoing cholecystectomy for acute Cholecystitis is hypotensive and tachycardic. Pulmonary capillary wedge pressure (PCWP) is elevated to 18 mmHg, and cardiac output is 3 L/min. she is shock best described as which of the following? A. hypovolemic shock B. septic shock C. cardiogenic shock D. anaphylactic shock (Classification -Recall; Source – Schwartz‟s Textbook of Surgery (8th edition): pp.) 27.

A 19 year old man is brought to the emergency department with a stab wound at the base of the neck (zone I) The most important concern for patients with such injuries is which of the following? A. upper extremity ischemia B. cerebral infarction C. exsanguinating hemorrhage D. mediastinitis (Classification -Recall; Source – Schwartz‟s Textbook of Surgery (8th edition): pp.) 28.

In septic shock, which of the following is true? A. the mortality rate is beteeen 10& and 20% B. gram-negative organisms are involved exclusively C. the majority of patients are elderly D. the most common source of infection is the alimentary tract. (Classification - Recall; Source – Schwartz‟s Textbook of Surgery (8th edition): pp.) 29.

The injury most often missed by selective nonoperative management of abdominal stabwounds is to which of the following? A. colon B. spleen C. ureter D. diaphragm (Classification - Recall; Source – Schwartz‟s Textbook of Surgery (8th edition): pp. ) 30.

A 63-year-old male tobacco smoker has a 1.5 cm non-healing ulcer in his lower lip. A nodular lesion is palpable deep to the ulcer. The most likely diagnosis is: A. squamous cell carcinoma B. keratoacanthoma C. malignant fibrous histiocytoma D. verrucous carcinoma (Classification - Application; Source – Schwartz‟s Textbook of Surgery (8th edition): p. 518)

31.

A 43-year-old-man presents with nasal obstruction and occasional epistaxis. Imaging studies combined with endoscopic biopsy have led to the diagnosis of nasopharyngeal cancer. The standard treatment is: A. intracavitary radiation B. external beam radiation C. combined chemotherapy and radiation D. surgical resection (Classification - Recall; Source – Schwartz‟s Textbook of Surgery (8th edition): pp. 1049-1050) 32. Biopsy of a slow growing infraauricular mass reveals a benign pleomorphic adenoma that is confined to the superficial lobe of the parotid gland. The standard treatment is: A. tumor enucleation B. tumor excision with 2 mm margin C. superficial parotidectomy D. total parotidectomy (Classification - Application; Source – Schwartz‟s Textbook of Surgery (8th edition): pp. 538-540) 33.

A 58 year old woman undergoes excision biopsy of a tumor in the left posterior triangle of her neck. Histology suggests that this is a metastatic cancer. What is the most likely site of the primary tumor? A. ovary B. adrenal gland C. kidney D. piriform fossa (Classification - Recall; Source – Schwartz‟s Textbook of Surgery (8th edition): pp.) 34.

A 59 year old woman has discomfort in the posterior part of her tongue. A biopsy confirms that the lesion is a carcinoma. What is true in carcinoma of the posterior third of the tongue? A. lymphoid tissue is absent B. lymph gland spread is often encountered C. there is an excellent prognosis D. the tissue is well differentiated (Classification - Recall; Source – Schwartz‟s Textbook of Surgery (8th edition): pp.) 35.

A 60-year-old man has a 3 cm nodular lesion with central ulceration in his left cheek. Section biopsy proves this to be basal cell carcinoma. This is best managed with: A. external beam radiation B. topical 5-fluouracil C. electrodessication D. excision with 2-4 mm margin (Classification - Application; Source – Schwartz‟s Textbook of Surgery (8th edition): pp. 439-440) 36.

This is the most common aggressive primary malignant bone tumor in adolescent and occurs in methaphyseal area of long bones with high incidence of pulmonary metastases: A. Chrodrosarcoma B. Osteosarcoma C. Fibrosarcoma D. Adamantinoma (Classification -Recall; Source – Schwartz‟s Textbook of Surgery (8th edition): pp.) 37.

A 65 year-old farmer presents with a 1.5-cm ulcerated lesion on the middle third of his lower lip. The lesion has been present for 4 months and is not painful. No lymph nodes are palpable in the patient‟s neck. The most likely diagnosis is: A. Squamous cell carcinoma B. Basal cell carcinoma C. Herpes simplex D. Keratoacanthoma (Classification – Application; Source - PRINCIPLES OF SURGERY by SCHWARTZ, SPENCER, SHIRES 7TH EDITION, Vol. 1, 521-522) 38.

A 65 year-old patient who spends summer in Baguio City presents with a painless, ulcerated lesion on the right cheek. The lesion has been present for one year. Physical examination of the patient‟s neck reveals no lymph node enlargement. The most likely diagnosis is: A. pyogenic granuloma B. melanoma C. basal cell carcinoma D. squamous cell carcinoma (Classification – Application; Source - PRINCIPLES OF SURGERY by SCHWARTZ, SPENCER, SHIRES 7 TH EDITION, Vol. 1, p.522) 39.

A 60-year-old woman has a 10-day history of cough and fever. Imaging studies show multiple fluid loculations in the right chest cavity with an estimated volume of 500 ml. Thoracentesis draws purulent material. The best treatment strategy now is: A. pigtail catheter drainage of the empyema B. video assisted thoracoscopic drainage with deloculation C. insertion of multiple thoracostomy tubes D. early thoracotomy and drainage (Classification - Application; Source – Schwartz‟s Textbook of Surgery (8th edition): pp. 601-602) 40.

A-50-year-old woman presents with a 3 cm solitary pulmonary nodule in the left upper lobe. She has a history of total thyroidectomy for papillary cancer at the age of 38 years. The main consideration is: A. TB granuloma B. hamartoma

C. primary lung cancer D. metastatic thyroid cancer (Classification - Application; Source – Schwartz‟s Textbook of Surgery (8th edition): pp. 556-557) 41.

A 45-year-old woman presents with weakness towards the afternoon, shortness of breath, and ptosis. CT scan reveals an anterior mediastinal mass. The most likely diagnosis is: A. intrathoracic goiter B. non-Hodgkin‟s lymphoma C. non-seminomatous germ cell tumor D. thymoma (Classification - Application; Source – Schwartz‟s Textbook of Surgery (8th edition): pp. 593-594) 42.

A man with malignant pleural effusion in association with an inoperable lung cancer may benefit from the instillation of sclerosing agent into the pleural cavity, once the lungs are almost fully expanded. An accepted sclerosant is: A. talc B. gentamicin C. penicillin G D. Vitamin C (Classification - Application; Source – Schwartz‟s Textbook of Surgery (8th edition): pp. 600-601) 43.

A 20-year-old woman consults for a 2-cm mass in her left breast. The mass is movable, non-tender and has a rubbery consistency and smooth borders. The probable diagnosis is: A. fibroadenoma B. fibrocystic disease C. carcinoma D. cystosarcoma phyllodes (Classification - Application; Source – Schwartz‟s Textbook of Surgery (8th edition): pp. 463-464) 44.

A 16-year-old girl is bothered by cyclical premenstrual pain in her breasts. She has 0.5-1.0 cm nodularities in the upper outer quadrant of both breasts. What management advice should be given to her? A. Observation B. breast ultrasound C. mammography D. aspiration biopsy (Classification - Application; Source – Schwartz‟s Textbook of Surgery (8th edition): pp. 465-466) 45.

An asymptomatic 40-year-old woman with no palpable breast mass undergoes mammography. This reveals clustered microcalcifications in the lower outer quadrant of her left breast. The probable diagnosis is: A. florid hyperplasia B. sclerosing adenosis C. atypical ductal hyperplasia D. ductal carcinoma in situ (Classification - Application; Source – Schwartz‟s Textbook of Surgery (8th edition): pp. 472-473) 46.

A 45-year-old woman with “lumpy” breasts undergoes mammography. Mammographic findings are suggestive of high-grade ductal carcinoma in situ confined to the upper outer quadrant of her right breast. How should she be managed? A. lumpectomy alone B. lumpectomy and radiotherapy C. mastectomy D. mastectomy and axillary node sampling (Classification - Application; Source – Schwartz‟s Textbook of Surgery (8th edition): p. 481)

47.

A 45-year-old woman has a recent onset bloody nipple discharge from her right breast. No palpable breast mass is noted. What diagnostic test is indicated? A. breast ultrasound B. mammography C. ductography D. cytology of discharge (Classification - Application; Source – Schwartz‟s Textbook of Surgery (8th edition): p. 493) 48.

A premenopausal woman undergoes modified radical mastectomy for a 3-cm breast cancer. No axillary node and distant metastases are detected. Test for this biomarker is currently recommended to facilitate the selection of adjuvant chemotherapy: A. c-fos B. c-myc C. p53 D.AHER2/neu (Classification - Application; Source – Schwartz‟s Textbook of Surgery (8th edition): p. 493) 49.

A 63-year-old man with chronic atrial fibrillation has sudden onset of pain, weakness, and paresthesia in his left leg, which appears cool, cyanotic, and without femoral and distal pulses. The right leg has normal pulses. The most likely diagnosis is: A. arterial embolism B. aortoiliac thrombosis C. Buerger‟s disease D. Raynaud‟s disease (Classification - Application; Source – Schwartz‟s Textbook of Surgery (8th edition): pp. 759-762) 50.

Coronary angiogram reveals a triple-vessel disease in a diabetic 55-year-old man presenting with unstable angina. The recommended treatment is: A. catheter-directed thrombolysis

B. balloon angioplasty C. coronary artery bypass grafting D. Transmyocardial revascularization (Class;ification - Application; Source – Schwartz‟s Textbook of Surgery (8th edition): pp. 652-653) 51.

A 41-year-old male chronic smoker has a 2-year history of bilateral foot claudication. He now has ulceration of the tip of the right 3rd toe and left 2nd and 4th toes. Popliteal pulses are palpable but the posterior tibial and dorsalis pedis pulses are bilaterally absent. The most important step in management is: A. cessation of smoking B. long-term anticoagulant therapy C. multiple toe amputations D. angiography followed by bypass surgery (Classification - Application; Source – Schwartz‟s Textbook of Surgery (8th edition): pp. 792-793) 52.

A premature infant with progressive dyspnea without cyanosis since birth has machinery murmur in the pulmonic area. A diagnosis of patent ductus arteriosus (PDA) is made. The indicated treatment is: A. administration of indomethacin B. infusion of prostaglandin C. administration of indomethacin D. immediate surgical correction of PDA (Classification - Recall; Source – Schwartz‟s Textbook of Surgery (8th edition): pp. 617-618) 53.

A 3-year-old girl presents with cyanotic spells that have increased in severity and frequency since birth. X-ray shows a boot-shaped heart; ECG reveals right ventricular hypertrophy. The most probable diagnosis is: A. truncus arteriosus B. tetralogy of Fallot C. transposition of the great arteries D. Ebstein‟s anomaly (Classification - Application; Source – Schwartz‟s Textbook of Surgery (8th edition): p. 636) 54.

A 54-year-old man presents with hematemesis after a bout of vomiting and retching. Endoscopy shows linear mucosal tears at the gastroesophageal junction. The diagnosis is: A. Mallory-Weiss syndrome B. Boerhaave‟s syndrome C. Menetrier‟s disease D. Dieulafoy‟s lesion (Classification - Recall; Source – Schwartz‟s Textbook of Surgery (8th edition): p. 919) 55.

After truncal vagotomy and antrectomy with gastrojejunostomy for an obstructed duodenal ulcer, the patient complains of diaphoresis, weakness, and abdominal discomfort followed by diarrhea several minutes after meals. The symptoms are suggestive of: A. post-vagotomy diarrhea B. bile reflux gastritis C. afferent loop syndrome D. dumping syndrome (Classification - Recall; Source – Schwartz‟s Textbook of Surgery (8th edition): pp. 985-987) 56.

A woman undergoes total gastrectomy for a huge proximal gastric carcinoma. To protect her from developing anemia, she must be given a regular parenteral dose of: A. folic acid B. ferrous sulfate C. vitamin B12 D. transferrin (Classification - Recall; Source – Schwartz‟s Textbook of Surgery (8th edition): p. 990) 57.

A few weeks after recovering from severe pancreatitis, a 34-year-old man has developed a pancreatic pseudocyst with mature wall that is pressing on his stomach. Aspiration fails to relieve the pressure symptoms. The indicated treatment is: A. administration of somatostatin B. internal drainage (cystogastrostomy) C. external drainage D. excision of pseudocyst (Classification - Application; Source – Schwartz‟s Textbook of Surgery (8th edition): pp. 1256-1258)

58.

ERCP reveals a periampullary cancer in a 64-year-old man admitted for jaundice. No metastasis is detected; no comorbid conditions are identified. How should he b managed? A. radical excision of the head of pancreas and duodenum B. local excision and adjuvant chemotherapy C. external beam radiation D. stenting and chemotherapy (Classification - Application; Source – Schwartz‟s Textbook of Surgery (8th edition): pp. 1281-1288) 59.

A 60 year old man with no previous operation has a 5-day history of inability to pass flatus or feces, cramping abdominal pain, and progressive abdominal distention. The principal diagnostic consideration is: A. midgut volvulus sigmoid volvulus B. colorectal cancer ileocecal tuberculosis (Classification - Application; Source – Schwartz‟s Textbook of Surgery (8th edition): pp.) 1.

A 65 year old male who underwent a colon resection of carcinoma 2 years prior to consult is found to have 3 solid nodules approximately 2 cms each in the right and left lobes of the liver. There is no extrahepatic disease detected. The preferred treatment is:

A. systemic chemotherapy B. radiofrequency ablation C. microwave coagulation therapy D. surgical resection of metastatic nodules (Classification – Application; Source – Schwartz‟s Textbook of Surgery (8th edition): pp.) 2.

A 35 year old female with a prolonged intake of contraceptive pills develops right upper quadrant pains. Ultrasound shows an isodense 5 cm mass in the right lobe of the liver. Sulfur-colloid scan done showed a cold lesion. This patient should undergo. A. conservative treatment B. resection C. enucleation D. liver transplantation (Classification - Application; Source – Schwartz‟s Textbook of Surgery (8th edition): pp. 119-120) 60.

A man who has an obstructed rectosigmoid cancer in association with a competent ileocecal valve is liable to develop perforation of: A. sigmoid colon B. splenic flexure C. hepatic flexure D. cecum (Classification - Application; Source – Schwartz‟s Textbook of Surgery (8th edition): pp. 1089-1090) 61.

The presence of tenesmus, decrease in caliber of stools and occasional bloody-mucoid diarrrrhea in a 67-year-old man is suggestive of: A. rectal carcinoma B. amebic colitis C. intussusception D. ileocecal tuberculosis (Classification - Application; Source – Schwartz‟s Textbook of Surgery (8th edition): pp. 1090-1091) 62.

Diagnostic tests show an ileocolic intussusception in a 38-year-old man, who has no signs of strangulation. What is recommended for this patient? A. expectant treatment B. endoscopic decompression C. hydrostatic reduction D. exploratory laparotomy (Classification - Application; Source – Schwartz‟s Textbook of Surgery (8th edition): pp.) 63.

A 63-year-old man is presently asymptomatic after medical treatment for LLQ pain and tenderness. He should undergo: A. CEA determination B. Colonoscopy C. barium enema D. abdominal CT scan (Classification - Application; Source – Schwartz‟s Textbook of Surgery (8th edition): pp.) 64.

A 56-year-old woman taking NSAID for chronic arthritis experienced severe epigastric pain 6 hours ago. Her abdomen is now diffusely tender with board-like rigidity. The appropriate initial diagnostic test is: A. chest upright x-ray B. plain abdomen supine x-ray C. abdominal ultrasound D. abdominal CT scan (Classification - Application; Source – Schwartz‟s Textbook of Surgery (8th edition): pp.) 65.

Despite 3 months of medical treatment for posterior midline anal fissure, the man‟s fissure-related anal pain during and after defecation has become excruciating. The appropriate treatment is: A. excision of fissure B. botulinum toxin injection C. anal divulsion D. internal sphincterotomy (Classification - Application; Source – Schwartz‟s Textbook of Surgery (8th edition): pp. 1103-1104) 66.

During surgery for a large right indirect inguinal hernia, the cecum is noted to form part of the wall of the hernia sac. The patient has this type of hernia: A. sliding B. Richter‟s C. interstitial D. Spigelian (Classification - Application; Source – Schwartz‟s Textbook of Surgery (8th edition): p. 1358) 67.

A previously asymptomatic 45-year-old obese woman consults for a non-tender swelling below her right inguinal ligament just lateral to the pubic tubercle. A warranted diagnostic procedure for this patient is: A. fine needle aspiration B. ultrasound C. MRI D. herniography (Classification - Application; Source – Schwartz‟s Textbook of Surgery (8th edition): pp. 1364-1366)

68.

A 54 year old male taking anticoagulant for the fast 4 weeks developed 3 day history of abdominal pain. On physical examination there was a palpable mass at the supraumbilical area with direct tenderness. Fothergill sign way positive. What is your impression? A. desmoid tumor B. rectus sheath Hematoma C. omental torsion D. omental infarction (Classification - Application; Source – Schwartz‟s Textbook of Surgery (8th edition): pp.) 69.

A 12 year old female complain of 1 day history of epigastric pain which shifted to the right lower quadrant associated with anorexia and vomiting. Physical examination revealed direct and rebound tenderness on the right lower quadrant. With the preoperative diagnosis of acute appendicitis, she was operated on. However, intraoperatively there was blood tinged peritoneal fluid and appendix was noted to be normal. The most probable cause of the patient condition is: A. mesenteric cyst B. omental torsion C. ruptured rectus sheath Hematoma D. mesenteric adenitis (Classification - Application; Source – Schwartz‟s Textbook of Surgery (8th edition): pp.) 70.

A 45-year-old is discovered to have a solitary 2-cm firm nodule in his right thyroid lobe. No associated symptoms are noted; family history is negative for thyroid cancer. Diagnostic work-up should start with: A. thyroid function tests B. thyroid ultrasound C. thyroid scan D. fine needle aspiration biopsy (Classification - Application; Source – Schwartz‟s Textbook of Surgery (8th edition): pp. 1413-1416) 71.

A 59-year-old man presents with 4-cm firm mass in the left lobe of the thyroid associated with a firm occipital mass that is 6 cm in its widest diameter, which he has neglected for the past 9 months. Family history is negative for thyroid malignancy. He probably has this type of thyroid cancer: A. papillary B. follicular C. medullary D. anaplastic (Classification - Application; Source – Schwartz‟s Textbook of Surgery (8th edition): pp. 1420-1421) 72.

A 49-year-old woman complains of central obesity with moonlike facies and hirsutism. Her ACTH is markedly elevated, which is suppressed by large dose of dexamethasone. The probable pathology is: A. adrenal adenoma B. adrenal carcinoma C. pheochromocytoma D. pituitary adenoma (Classification - Application; Source – Schwartz‟s Textbook of Surgery (8th edition): pp. 1455-1458) 73.

A 42-year-old man presents with headache, palpitations, and diaphoresis. His urinary metanephrines are elevated. Imaging studies reveal a 3-cm mass in his right adrenal gland. The treatment of choice is: A. adrenalectomy B. radiotherapy C. tumor embolization D. chemoradiation (Classification - Application; Source – Schwartz‟s Textbook of Surgery (8th edition): pp. 1460-1461) 74.

A 67-year-old man is bleeding excessively following a transurethral prostatectomy. The bleeding is assessed to be due to local plasminogen activation leading to increased fibrinolysis on the raw wound surface. Administration of this drug may be beneficial: A. vitamin K B. heparin C. warfarin D. EACA (Classification - Recall; Source – Schwartz‟s Textbook of Surgery (8th edition): p. 82) 75.

A 60-year-old man presents with slight urinary urgency, frequency, and a decrease in the force of micturition. Digital rectal examination detects no mass in the prostate. The indicated diagnostic test is: A. prostatic acid phosphatase B. prostate-specific antigen C. pelvic ultrasound D. needle biopsy of prostate (Classification - Application; Source – Schwartz‟s Textbook of Surgery (8th edition): pp. 1535-1537) 76.

A 35-year-old man presents with right flank pain and microscopic hematuria. Intravenous pyelography reveals a 4 mm radiolucent stone in the distal right ureter. The recommended treatment is: A. alkalinization of the urine B. extracorporeal shock wave lithotripsy C. stone extraction via ureteroscope D. open ureterolithotomy (Classification - Application; Source – Schwartz‟s Textbook of Surgery (8th edition): pp. 1546-1549) 77.

A 55-year-old man is admitted for gross hematuria associated with left flank pain and anemia. CT scan detects a solid, enhancing mass in the left kidney. The likely diagnosis is: A. renal cell carcinoma

B. renal tuberculosis C. renal sarcoma D. renal oncocytoma (Classification - Application; Source – Schwartz‟s Textbook of Surgery (8th edition): pp. 1531-1532) 78.

A 23-year-old woman sustained blunt head trauma to her right frontoparietal area in a car crash, which caused he to be unconscious for several minutes. Upon arrival at the E.R., she is alert and has no neurologic deficit. X-ray shows a linear non-depressed right frontoparietal fracture. An hour after admission, vomiting followed by progressive neurologic deterioration are noted. A. diffuse axonal injury B. cerebral contusion C. acute epidural hematoma D. acute subdural hematoma (Classification - Application; Source – Schwartz‟s Textbook of Surgery (8th edition): p. 1619) 79.

A 54-year-old woman with no previous neurologic disorder has new onset seizures without associated gross neurologic deficit. She was treated for breast cancer 10 years ago. If brain metastasis is the main consideration, this is best confirmed by means of: A. electroencephalography B. CT scan of the brain with intravenous contrast C. MRI of the brain with intravenous contrast D. Doppler ultrasound (Classification - Recall; Source – Schwartz‟s Textbook of Surgery (8th edition): p. 1633) 80.

A 55-year-old man with a history of lung cancer presents with frequent severe headache, occasional vomiting, and weakness of the right upper extremity. What is the recommended treatment in addition to corticosteroids? A. intrathecal chemotherapy B. whole-brain radiotherapy C. stereotactic radiosurgery (gamma knife) D. resection of lesion causing arm weakness (Classification - Application; Source – Schwartz‟s Textbook of Surgery (8th edition): p. 1633)

81.

A young man just admitted for blunt injuries from a car accident is stuporous. CT scan shows cerebral contusion. What should be avoided as it can cause secondary brain injury to this patient through its deleterious effect on intracranial pressure? A. elevation of the head B. respiratory acidosis C. hypovolemia even of mild degree D. sedation of agitated patient (Classification - Recall; Source – Schwartz‟s Textbook of Surgery (8th edition): pp. 1613-1614) 82.

As a basketball player falls hard on his extended right arm, he feels immediate severe pain with subsequent inability to move his right arm. Swelling and tenderness about the shoulder with loss of deltoid contour are noted. The likely diagnosis is: A. deltoid muscle rupture B. shoulder dislocation C. fracture of proximal humerus D. brachial plexus injury (Classification - Application; Source – Schwartz‟s Textbook of Surgery (8th edition): pp. 1706-1707) 83.

A 10-week-old baby with cleft lip and palate has undergone a successful cleft lip repair. It is recommended that cleft pate repair be done before the baby reaches this age: A. 6 months B. 12 months C. 2 years D. 4 years (Classification - Recall; Source – Schwartz‟s Textbook of Surgery (8th edition): pp. 1797-1799) 84.

A bedridden 61-year-old woman has a significant neurologic recovery from a previous stroke. She has an infected deep pressure ulcer in her sacrum. Once the infection has been controlled by debridement and supportive treatment, the pressure ulcer can be managed with: A. skin graft B. rotation skin flap C. gluteus maximus flap D. microsurgery free flap (Classification - Recall; Source – Schwartz‟s Textbook of Surgery (8th edition): pp. 1825-1826)) A small thick antecubital scar from a 2 nd degree flame burn is constricting movement. To relieve the constriction, the appropriate procedure to refashion the scar is: A. Z-plasty B. advancement flap C. scar excision and skin graft D. deltopectoral flap (Classification - Recall; Source – Schwartz‟s Textbook of Surgery (8th edition): pp. 1792-1793)) 85.

86.

Resection of a sarcoma results in a significant lateral chest wall defect. To minimize pulmonary dysfunction, chest wall reconstruction may utilize: A. rectus abdominis muscle flap B. omental flap C. tensor fascia lata graft D. Marlex mesh

(Classification - Recall; Source – Schwartz‟s Textbook of Surgery (8th edition): pp. 1823-1824)) 87.

A 4-week-old baby boy presents with frequent non-bilious vomiting. During feeding, peristaltic waves are seen in the upper abdomen in association with a palpable mass in the right upper quadrant. The diagnosis is: A. hypertrophic pyloric stenosis B. duodenal atresia C. jejunal atresia D. volvulus neonatorum (Classification - Application; Source – Schwartz‟s Textbook of Surgery (8th edition): pp. 1486-1487) 88.

A newborn infant has excessive drooling followed by coughing immediately after feeding. The main diagnostic consideration is esophageal atresia. This can be readily confirmed by: A. inability to pass orogastric tube into the stomach B. water-soluble contrast study (esophagram) C. transesophageal ultrasound D. plain abdominal radiograph (Classification - Application; Source – Schwartz‟s Textbook of Surgery (8th edition): pp. 1481-1482)) 89.

A diagnosis of intussusception is made in an infant with paroxysms of crampy abdominal pain and vomiting. The infant has no peritoneal signs and remains hemodynamically stable. The initial treatment of choice is: A. nasogastric decompression B. endoscopic reduction C. pneumatic reduction D. hydrostatic reduction with barium (Classification - Recall; Source – Schwartz‟s Textbook of Surgery (8th edition): pp. 1493-1494) 90.

A 3-day old infant who has not passed meconium presents with abdominal distention and bilious vomiting. The most likely diagnosis is: A. duodenal atresia B. malrotation C. midgut volvulus D. Hirschprung‟s disease (Classification - Application; Source – Schwartz‟s Textbook of Surgery (8th edition): pp. 1496-1497) 91.

A 3-week-old infant is noted to have jaundice in association with acholic stools shortly after birth. A radionuclide scan demonstrates presence of radioisotope in the intestine. This finding rules out the diagnosis of: A. neonatal hepatitis B. biliary atresia C. inspissated bile syndrome D. choledochal cyst (Classification - Application; Source – Schwartz‟s Textbook of Surgery (8th edition): pp. 1499-1500) 92.

A 67 y/o man has an intraabdominal abscess caused by perforated sigmoid diverticulitis. The most common isolated microorganism is__? A. Escherichia coli B. Bacteroides C. K. pneumoniae D. Staphylococcus epidremides Schwartz‟s: surgical infection pp 121 93.

One of the most common complication of central venous access is_____. A. hemorrhage B. pneumothorax C. wound infection D. air embolus Schwartz‟s Complication in Surgery p.337 94.

Early postoperative small bowel obstruction is a rare finding and occurs less than 1 % of the time. When it does, the most common cause is_____. A. internal hernias B. post-operative adhesions C. infections or abscess D. technical errors Schwartz‟s: surgical complications p. 347 95.

Which of the ff statements is true concerning corrosive injury to the esophagus? A. Acid ingestion is not injurious to the stomach because of its non-acidic pH. B. Ingested caustic agents rapidly pass through the esophagus and stomach into the small intestines. C. Alkaline injury is more destructive than acid injury. D. Acid injury is more destructive than alkaline injury. Schwartz‟s: Esophagus p. 909 96.

A 27 year old female was brought to the ER with history of suicidal ingestion of household muriatic acid. This patient is best managed by--A. Give half strength vinegar, lemon juice or orange juice B. Start with milk, egg white or antacids C. Give emetics to induce vomiting D. Sodium bicarbonate is one of the treatment option. Schwartz‟s: Esophagus p. 910 97.

Which of the following statements is true regarding the arterial blood supply of the stomach?

A. B.

The right gastric artery, a branch of the superior mesenteric artery supplies the gastric antrum Because of the rich intramural collateral vessels, gastric viability may be preserved after ligation of at least two of the four named gastric arteries C. The largest artery supplying the stomach is the right gastric artery. D. The left gastroepiploic artery is a branch of the celiac trunk. Schwartz‟s: Stomach pp. 935-937 98.

At a cellular level, the major stimulants of acid secretion by the gastric parietal cell are the following, except: A. histamine B. prostaglandinE2 C. acetylcholine D. gastrin Schwartz‟s: Stomach p. 943 99.

A patient with gastric adenocarcinoma underwent subtotal gastrectomy. Histopath revealed tumor penetrating the serosa, regional lymph nodes are not involved and distant metastatic lesions are not detected. What is the correct tumor stage on TNM staging? B A. Stage I B. Stage II C. Stage III-A D. Stage III-B Schwartz‟s: Stomach p. 970 100.

A 5-month old baby boy was seen at the ER with complaints of non-bilous vomiting that became increasingly projectile over several days to weeks and cannot tolerate liquid intake. Abdominal examination revealed palpation of typical “olives” in the right upper quadrant with visible gastric waves. What is the correct diagnosis? A. Meconium ileus C. Hypertrophic pyloric stenosis Hirschprung‟s disease D. Intussusception Schwartz‟s: Pediatric Surgery p.1486 101.

Which of the following is the most common malignant liver tumor in childhood? A. Hemangioma C. Hepatocellular carcinoma B. Hepatoblastoma D. Mesenchymal hamartoma Schwartz‟s: Pediatric Surgery p.1512 102.

What is the appropriate treatment for an incidental 3 cm liver hemangioma found on abdominal CT scan of 37 year old woman? A. No treatment C. Selective embolization of left hepatic artery B. Cryoablation D. Left hepatic lobectomy Schwartz‟s: pp. 1162 – 1163 103.

A 35 y/o male presents with 1 year history of early satiety and right upper abdominal discomfort. CT scan shows a 6 x 8 cm cystic lesion in the right lobe of the liver. Ultrasound of cyst show no internal echoes. The appropriate management is: A. cryoablation of cyst C. unroofing of the cyst B. right hepatic lobectomy D. percutaneous aspiration of cyst Schwartz‟s: p. 1159 104.

An imaginary line that divides the liver into right and left lobe that runs from the inferior vena cava to the tip of the gallbladder fossa: C A. Catlie line B. Calot‟s line C. Cantlie‟s line D. Charcot‟s line Schwartz‟s: p. 1140 105.

The diagnostic tool important in evaluating liver injury in a stable blunt trauma patient is: A. Ultrasound C. CT scan B. Abdominal x-ray D. Peritoneal lavage Schwartz‟s: p. 142 106.

The local factor affecting wound healing: A. Age B. Smoking Schwartz‟s: p. 235

C. Low oxygen tension D. Trauma

107.

The characteristic of Keloid scars: A. Arise above the skin level and stay in the confine of the original wound B. Develop within 4 weeks after trauma C. Extend beyond the border of original wound and rarely regress spontaneously D. Occur in areas of flexion and extension Schwartz‟s: p. 240 108.

A 55 y/o male diagnosed to have chronic pancreatitis, developed epigastric mass measuring about 6 cm accompanied by pain and fullness. The diagnosis is: A. Gastric carcinoma C. Pseudocyt of the pancreas B. Pancreatic carcinoma D. Acute pancreatitis Schwartz‟s: p. 1256

109.

Type of benign solitary pancreatic neoplasm consisting of symptomatic fasting hypoglycemia and profound syncopal episodez: A. Insulinoma C. Gastrinoma B. Glucagoma D. VIPoma Schwartz‟s: p. 1275 110.

The most common indication for splenectomy: A. Staging of Hodgkin‟s disease C. Trauma to spleen B. Hereditary spherocytosis D. Thalassemia Schwartz‟s: p. 1301 111.

The best time to repair of cleft lip is: A. 10 months B. 10 weeks Schwartz‟s: p 1797

C. 6 months D. 16 weeks

112.

A 36 y/o female complain of an acute pain at the lateral aspect of her left breast. A tender firm cord is found on the same site on physical examination. It was diagnosed as a Mondors disease. This lesion is best managed with: A. Radiotherapy B. Anti-inflammatory medication, warm compress and rest of ipsilateral extremity C. Immediate excision of the lesion is needed D. Total mastectomy Schwartz‟s: p. 463 113.

Breast lesion that can be treated with close observation with or without tamoxifen: early invasive breast carcinoma lobular carcinoma in situ ductal carcinoma in situ inflammatory carcinoma of the breast Schwartz‟s: p.481 114.

The best screening imaging technique for breast cancer is: A. Magnetic resonance imaging B. Ultrasonography C. Ductography D. Mammography Schwartz‟s: pp. 476-477

D

115.

A 46 y/o female who underwent a total thyroidectomy for stage II papillary carcinoma developed 3 weeks later with Chvostek‟s and Trouseau‟s sign. This could be due to: A. Recurrence of papillary thyroid carcinoma B. Injury to cervical sympathetic trunk C. Hypoparathyroidism D. Injury to recurrent laryngeal nerve Schwartz‟s: pp. 1429-1448 116.

This is considered as a single most important test in the evaluation of patients with thyroid masses: A. Thyroid ultrasound B. CT and MRI C. Fine needle aspiration cytology D. Thyroid scanning Schwartz‟s: p. 1415 117.

Lymph node in the posterior triangle of the neck is within what level: A. Level II B. Level IV C. Level V D. Level VI Schwartz‟s: p. 534 118.

This form of shock has a low blood pressure, low urine output but has an elevated central venous pressure: A.Vasodilatory shock B. Obstructive shock C. Neurogenic shock D. Traumatic shock Schwartz‟s: p. 95-102 55 y/o male who has been in the ward for the past 4 days and is receiving IV fluid of D5W is complaining on his 5 th hospital day of headache, nausea, vomiting, anorexia, body malaie and watery diarrhea. Give the most probable electrolyte imbalance the patient might have: A. Hypernatremia B. Hyperkalemia C. Hyponatremia D. Hypokalemia Schwartz‟s: p. 48 119.

120.

The most important part of the treatment of severe metabolic acidosis among trauma patient is: A .Correct abnormality with exogenous bicarbonate B. High oxygenation E. Restore perfusion with volume resuscitation F. D. Give colloid solution Schwartz‟s: p. 50-51

121.

The most common primary bone malignancy: A. Osteosarcoma B. Ewing‟s sarcoma C. Ostoblastoma D. Chondrosarcoma Schwartz‟s: p. 1665 122.

The most potent stimulant for aldosterone release is: A. ACTH B. rennin C. angiotensin I D. angiotensin II Schwartz‟s: p. 9 123.

The most biologically active and potent vasoconstrictor is angiotensin II endothelins epinephrine serotonin Schwartz‟s: p. 23 124.

In trauma patients significant reduction in infectious complications were noted in patients given early enteral nutrition as compared to those given who are unfed or given parenteral nutrition except for what type of trauma patients? A. renal trauma B. long bone fractures C. head trauma D. liver trauma Schwartz‟s: p. 31 125.

Branched chain amino acids are used in parenteral nutrition for what specific type of patients? A. renal failure patients B. diabetic patients C. pulmonary failure patients D. hepatic failure patients Schwartz‟s: p.33 126.

In what type of parenteral feeding is fat increased to 50% of total calories? A. Heart failure formulas B. pulmonary failure formulas C. renal failure formulas D. calorie-dense formulas Schwartz‟s: p. 33 127.

The appendeceal flora remains constant throughout life with the exception of what bacteria, which appears only in adults? A. Porphyromonas gingivalis B. Pseudomonas aeruginosa C. Peptostreptococci D. Streptococcus anginosus Swhartz‟s: p. 1121 128.

The most significant factor associated with both fetal and maternal deaths in pregnant patients with acute appendicitis is A. formation of a phlegmon B. appendeceal perforation C. appendeceal tip near the uterus D. retrocecal appendix Schwartz‟s: p. 1129 129.

Which of the ff. is true of the superior mesenteric artery syndrome? A. It is often seen in obese individuals. B. It usually affects patients over 70 yrs. old. C. Compression is over the third portion of the duodenum. D. It is a surgical emergency. Swhartz‟s: p. 1027

C

130.

Which of the following factors present in a entero-cutaneous fistula increases the possibility of closure of the tract? A. non-epithelialization of the tract B. high output fistula C. malnutrition D. sepsis Swhartz‟s: p. 1037 131.

What is the single most important factor in predicting burn related morbidity and mortality? A. size of burn B. type of burn C. etiology of burn D. associated medical condition Schwartz 7 th ed., p. 228-232 / Ans. A

132.

What type of shock is caused by the interference of the balance of vasolidator and vasoconstrictor influences to arterioles and venules? A. cardiogenic shock B. hypovolemic shock C. septic shock D. neurogenic shock Shwartz 7th ed., p. 113-115 / Ans. D 133.

A 50 y/o female consulted to you and complaints of fatigue, polydypsia, polyuria, nocturia, joint pains and constipation. Her laboratory exams shows normal FBS, elevated serum calcium, and elevated intact PTH. What would be the most likely diagnosis? A. NIDDM B. Metastatic breast ca C. Hyperthyroidism D. Hyperparathyroidism Shwartz 7th ed., p. 1434-1439 / Ans. D 134.

A 35 y/o male presents with a long standing severe hypertension with associated muscle weaknes, headache, polyuria and polydypsia. Blood exams revealed an elevated aldesterone level and elevated serum potassium. What is the most likely diagnosis? A. cushing‟s syndrome B. pheochromocytoma C. conn‟s syndrome D. waterhouse-friderichsen syndrome Shwartz 7th ed., p. 1453-1458 / Ans. C 135.

Which of the following is the most commonly inured organ in blunt abdominal injury? A. small intestine B. large intestine C. pancreas D. spleen Schwartz 7 th ed., p. 187 / Ans. D 136.

In the initial management of an acutely and seriously injured patient, the first and most important emergency measure to be taken care of is: A. splinting of fracture B. control of bleeding C. restoration of blood volume D. ensuring an adequate airway Schwatz 7th ed., p. 156 / Ans. D 137.

Diagnostic peritoneal lavage (DPL) remains the most sensitive test available for determining the presence of intraabdominal injury. The result of DPL is considered grossly positive if; A. 1 ml of blood is aspirated B. 5 ml of blood is aspirated C. 8 ml of blood is aspiarated D. 15 ml of blood is aspirated Schwartz 7 th ed., p. 167 / Ans. D 138.

Which of the following constitute an immediate threat to life because of inadequate ventilation A. flail chest B. hemothorax C. stabbed wound involving the diaphragm D. pulmonary hematoma Schwartz 7 th ed., p.156-157 / Ans. A 139.

While undergoing exploratory laparotomy for blunt abdominal injury, to the patients BPwas noted to be 70 palpatory while bleeding was noted coming from the splenic hilum. Appropriate management for the injured spleen should be; A. splenectomy B. splenorrhapy C. debridement and repair D. packing, immediate closure of abdominal lesion and volume replacement Schwartz 7 th ed., p. 193 / Ans. A 140.

During the initial resuscitation of a hypovolemic patient secondary to a vehicular accident external bleeding is best controlled by; A. suturing B. direct finger pressure C. application of tourniquet D. blood transfusion Schwartz 7thed., p. 157 / Ans. B 141.

In penetrating neck injuries zone II is referred to an area between; A. clavicle and cricoid cartilage B. cricoid cartilage and angle of mandible C. above the angle of the mandible D. below the clavicle Schwartz 7 th ed., p. 165 / Ans. B

142.

In assessing the extent of hepatic injury following blunt injury to the abdomen, bleeding from the liver is best controlled by; A. manual cmpression of liver parenchyma B. using figure of 8 - suture C. pringle maneuver D. liver resection Schwartz 7 th ed., p. 188-189 / Ans. C 143.

Stable patients at risk of urethral injury, manifestating as presence of blood at the matus, necessitates an immediate; A. urethral catheterization B. urethrography C. cystoscopy D. KUB-IVP Schwartz 7 th ed., p. 161-162 / Ans. B 144.

Type of healing in which the wound is allowed to heal by granulation tissues formation and contraction: A. primary intention B. secondary intention C. tertiary intention D. quarternary intention Schwartz 8 th ed., p. 234 / Ans. B 145.

Partial thickness wound such as seen in superficial second degree burns heal by which of the following process: A. wound contraction B. epithelization C. granulation tissue formation D. maturation and remodeling Schwartz 8 th ed., p. 228 / Ans. B 146.

Equilibrium between collagen synthesis and collagen degradation occurred during the stage of; A. hemostasis and inflammation B. proliferative stage C. matrix synthesis D. remodeling Schwartz 8 th ed., p. 228 / Ans. D 147.

Delayed primary wound closure is indicated in which of the following type of wound; A. diabetic ulcer B. decubitus ulcer C. contaminated traumatic wound D. surgical incision following simple appendectomy Schwartz 7 th ed., p. 264 / Ans. C 148.

Cytokines that are produced by one cell and affect an adjacent cell such as transforming growth factor beta (TGFB) are called; A. endocrine factors B. paracrine factors C. autocrine factors D. intacrine factors Schwartz 7 th ed., p. 266 / Ans. B 149.

The major component of the extracellular matrix that provide strength, support and structure of all soft tissues, tendons, ligaments and bones is; A. collagen B. elastin C. fibronectin D. hyaluronic acid Schwartz 7 th ed., p. 270-271 / Ans. A 150.

The genetic disorder arising from mutations in the genes for type I collagen causing increase propensity for the bones to break under minimal stress is; A. ehlers-danlos syndrome B. osteogenesis imperfecta C. marfan‟s syndrome D. epidermolysis bullosa Schwartz 7 th ed., p. 278 / Ans. B 151.

Lesion of the gastrointestinal tract that result in comlete regeneration and recovery; A. peptic ulcer disease B. crohns disease C. radiation colitis D. gastric erosions Schwartz 7 th ed., p. 280-281 / Ans. D 152.

Skin lesions that extend beyond the boundaries of the original wound, do not regress with time and recur after excision; A. hypertrophic scars B. keloids C. diabetic ulcers D. decubitus ulcers Schwartz 7th ed., p. 281-282 / Ans. B

153.

Kehr‟s sign is a classic example of; A. unreferred visceral pain B. referred visceral pain C. unreferred parietal pain D. referred parietal pain Scwratz 7th ed., p. 1034-1035 / Ans. D 154.

The bleeding in Mallory-Weiss syndrome is secondary to; A. penetration of ulcer affecting the gastroduodenal artery B. linear tears of the gastroesophageal junction C. gastric erosions D. hematobilia Schwartz 7 th ed., p. 1062-1063 / Ans. B 155.

The most common cause of intestinal obstruction for all age group combined is; a. trangulated hernia b. volvulus c. adhesive bands d. neoplasm Schwartz 7 th ed., p. 1054 / ans. C

156.

The most obvious route of fuid and electrolyte loss in patients with intestinal obstruction from leocecal TB is through; the edematous bowel wall the intestinal lumen vomiting the peritoneal cavity Schwartz 7 th ed., p . 1056 / Ans. C 157.

Occlusion of the blood supply to a segment of bowel in addition to obstruction of the lumen is referred to as; A. complete intestinal obstruction B. closed-loop obstruction C. strangulated obstruction D. ileus Schwartz 7 th ed., p. 1057 / Ans. C 158.

Which of the following diagnostic modalities consistently localizes the site of bleeding in cases of lower GI bleeding; A. colonoscopy B. CT scan C. Angiography D. Barium enema Schwartz 7 th ed., p. 1066 / Ans. C 159.

The most common cause of massive lower GI bleeding is; A. colonic malignancy B. diverticulosis C. adenamatous polyps D. ulcerative colitis Schwartz 7 th ed., p. 1065 / Ans. B 160.

A 50 y/o male with history of alcoholism was admitted because of history of progressive jaundice, associated with tea-colored urine, achlic stools, weight loss and mild epigastric pain. The simplest and most non-invasive method in the diagnosis of this patient is; A. percutaneous transhepatic cholangiography (PTC) B. ERCP C. Ultrasonography D. HIDA scan Schwartz 7 th ed., p. 1071 / Ans. C 161.

Relative to the ileum, the jejunum has; A. more fatly mesentery B. less prominent plicae circularis C. longer vasa recta D. lesser in diameter Schwatz 8th ed., p . 1018 / Ans. C

162.

The most common malignant neoplasm of the small bowel is; adenocarcnoma carcinoids sarcomas lymphomas Schwartz 7 th ed., p. 1242 / Ans. A 163.

The presence of a Meckels diverticulum in a hernial sac is called; A. spingelian hernia B. femoral hernia C. littre‟s hernia D. richter‟s hernia Schwartz th ed., p. 1249 / ans. C 164.

Rovsings‟ sign is elicited by; A. pain the right lower quadrant when palpatory pressure is exerted in the left lower quadrant B. pain in the right lower quadrant when palpatory pressure is exerted directly over the pont of tenderness C. flexion-extension of the right lower extremity D. asking the patient to cough Schwartz 7 th ed., p. 1385 / Ans. A 165.

In children with history of URTI 3 days prior to development of RLQ pain, the differential diagnosis most often confused with appendicitis is; A. meckel‟s diverticulitis B. perforated peptic ulcer C. acute gastroenteritis D. acute mesenteric adenitis Schwartz 7 th ed., p. 1387 / Ans. D 166.

The recommended treatment for patients with adenocarcinoma of the appendix is; A. appendectomy B. right hemicolectomy C. total colectomy D. subtotal colectomy Schwartz 7ht ed., p. 1392 / Ans. B 167.

The precipitating factor in secondary omental torsion is; A. bifid omentum B. accessory omentum C. obesity D. foci of intraabdominal inflammation Schwartz 7 th ed., p. 1556 / Ans. D 168.

The most common solid tumor of the omentum is; A. lymphoma B. liposarcoma C. leiomyosacoma D. metastatic carcinoma Schwartz 7h ed., p. 1558 / Ans. D 169.

The most common anaerobic bacteria that is isolated in intraabdominal infection is; A. clostridium specios B. peptococcus specios C. bacteroides specios D. fusobacterium specios Schwartz 7 th ed., p. 1524-1525 / Ans. C 170.

A 45 year old female came in because of right upper quadrant pain, colicky aggravated by fatty food intake 4 hours ptc. Physical exam showed a palpable mass at the right subcostal margin non-movable tender, midinspiratory arrest was illicited while the palpating hand was on the right subcostal area . the most likely condition is/are A. acute appendicitis B. acute cholecystitis C. perforated peptic ulcer D. acute ascending cholangitis Schwartz / Ans. B 171.

The laboratory examination of a 45 year old female came in with the following results ; sgpt was elevated, direct bilirubin increased, alkaline phosphatase increased, protime prolonged . utz showed a hyperechoic mass at he neck of the gallbladder with posterior shadowing. The most likely diagnosis is A. acute appendicitis B. acute infectious hepatitis C. acute calculous cholecystitis D. ascending cholangitis Schwartz / Ans. C 172.

The treatment of choice for the above condition ( no.4) is A. appendectomy B. medical management C. cholecystectomy D. ercp Schwartz / Ans. C

173.

A patient s/p cholecystectomy and cbde with t-tube choledochostomy showed a distal cbd stone on t-tube cholangiography , management would be A. daily irrigation with nss B. ercp C. choledococopy and basket extraction of the stone D. repeart explore lap and cbde Schwartz / Ans. C

174.

A 25 year old male came in with chronic draining lesion at the perianal area of 2 years duration. Physical exam showed a draining lesion about 4 cm from the anal verge at the 3:00 position. The internal opening is probably A. anterior midline B. posterior midline C. radially directly opposite D. variable Schwartz / Ans. B

175.

30 year old female came in because of painfull perianal mass 2 days ptc, physical exam showed a tender mass at the right lateral position w/c precludes rectal exam with a bulging tender mass adjacent to it. The most likely condition is A. thrombosed external hemorrhoids B. prolapsed internal hemorrhoid C. perianal abscess D. rectal polyp Schwartz / Ans. C

176.

A 50 year old male came in because of loss of weight about 30% of his previous body weight. Associated with on and off colicky abdominal pain. He also noticed that his stools have diminish in caliber just like a goat stool,the single most important examination would be A. fecalysis B. ba enema C. colonoscopy w/ biopsy D. ct scan Schwartz / Ans. C

177.

A 25 year old female came in with a palpable breast mass noted 2 weeks ptc, the mass is about 2.5 cm well delineated border, movable, non-tender at the right upper outer quadrant, no familial history of ca.the most likely diagnosis would be A. fibroadenoma B. fibrocystic disease C. breast ca D. galactocoele Schwartz / Ans. A 178.

Seromas are fluid collections coming from the: A. Plasma B. Interstitium C. Lymph D. Capillary oozing

179.

A patient was discharged 3 days after appendectomy (suppurative). After 5 days, he came back at the OPD for follow up complaining of tenderness at the operative site. The incision site was nonerythematous. What is your diagnosis? A. Hyperesthesia B. Surgical site infection C. Foreign body reaction to suture D. Urinary tract infection

180.

A patient was operated on for acute cholecystitis. After 24h postop, the patient developed fever. What is the most likely cause of his fever? A. Cathether sepsis B. Drug-related conditions C. Aspiration pneumonia D. Atelectasis

181.

The basic problem in poor wound healing regardless of the underlying factor is: A. Poor local hemostasis B. Anemia C. Low 02 tension D. Impaired inflammatory response

182.

Ileus following A. B. C. D.

183.

For every degree rise in temperature, the insensible water loss in cm3 per day is approximately: A. 100 B. 150 C. 200 D. 250

abdominal surgery is expected to last for at least how many days? 3 4 5 6

184.

Which ion if altered determines the shift of fluid from one compartment to another? A. Sodium B. Potasium C. Chloride D. Bicarbonate

185.

Which of the following cytokines may help control keloids and hypertrophic scars? A. Pdgf B. Egf C. TNF D. TGB-B

186.

Which part of the GIT provides strength in the anastomosis A. Serosa B. Muscularis C. Submucosa D. Mucosa

187.

Which amino acid provide energy source in the GIT A. Alanine B. Valine C. Glutamine D. Tryptophan

188.

A 54yo male patient with 3rd degree burn approximately 40%TBSAdeveloped abdominal pain on the 4 th hospital day. What is the least likely cause of the abdominal pain? A. Acalculous cholecystitis B. Acute pancreatitis C. Superior mesenteric artery syndrome D. Renal colic

189.

Convulsions if A. B. C. D.

190.

The offending organism in surgical site infection is; A. Staph aureus B. Pseudomonas C. E. Coli D. Bacteroides

191.

Earliest manifestations of catheter related complication following parenteral nutrition. A. Tachycardia B. Fever C. Glucose intolerance D. Changes in sensorium

192.

Which of the following statements is not a sound principle in the fluid and electrolyte therapy post operatively? A. Thorough evaluation of the pre and intra-op fluid status B. Urine volume is replaced on a mililiter to mililiter basis C. It is not necessary to give potassium with in the first 24 hour D. Insensible water loss is considered in the fluid therapy

193.

The average potassium (in meq/l) content of the bile per day is A. 5 B. 10 C. 15 D. 18

194.

Which cranial nerve is involved in the act of swallowing A. Xi B. X C. Xi D. Viii

195.

Primary reason A. B. C. D.

196.

Most common A. B. C. D.

197.

Blood supply of lesser curvature is

present during the resucitative phase of burn injury may be due to Hypoxemia Hyponatremia Infection Hypokalemia

for staging esophageal cancer is Determine its resectability Prognostication To assess whether the procedure is for cure or palliation Whether pre-op chemotherapy is indicated

benign esophageal tumor is Leiomyomas Fibromas Myomas Fibromyomas

A. B. C. D.

Left gastric Right gastric Gastroepiploic Splenic

198.

Manifestation of acute gastric dilatation A. Pallor B. Rapid respiration C. Changes in sensorium D. Hypotension

199.

A patient underwent upper gastrointestinal endoscopy for gastric outlet obstruction. The endoscopist noted a lesion at the antrum and took biopsy. It turned out to be gastric cancer. What type of gastric cancer has favorable prognosis? A. Superficial spreading B. Polypoid C. Ulcerative D. Linitis plastica

200.

A patient diagnosed to have perforated peptic ulcer disease had his symptoms 8 hours prior to admission. He was prepared for surgery. Which of the following best describes the required operation? A. Repair of perforation and selective vagotomy B. Repair of perforation and truncal vagotomy C. Suture of perforation D. Omental patch

201.

After truncal vagotomy for peptic ulcer disease, the patient may develop the following except: A. Gastric stasis B. Megaloblastic anemia C. Diarrhea D. Gallstone formation

202.

Most common A. B. C. D.

203.

A 77 year old, male, chronic smoker on NSAIDs for arthritis presents with an acute abdomen and pneumoperitoneum. What could be the most likely cause of this? A. Severe abdominal pain due to chronic gastritis B. Acute pancreatitis C. Acute cholecystitis with cholangitis D. Perforated peptic ulcer disease

204.

Increased venous pressure , decreased pulse pressure and decrease heart sound are pathognomonic of: A. Acute myocardial infarction B. Pneumothorax C. Cardiac tamponade D. Aneurysm of the arch of aorta

205.

“Bird‟s beak” deformity is a radiologic sign of A. Sigmoid volvolus B. Ileoascending intussuception C. Sigmoid carcinoma D. Cardiac tamponade

206.

A 32 yo male was brought to the ER because of blunt abdominal trauma due to fall. The patient had hematoma and abrasions on his RUQ and R flank. Your resident assessed that the patient has surgical abdomen and needs immediate surgery. He requested one-shot intravenous pyelography en route to or. The reason for such procedure is to: A. Find out extravasation of urine B. Localize site of obstruction C. Determine function of either kidneys D. Quantify volume of urine output

207.

A 23 yo female, medical student, was brought to the ER because of VA. Patient was GCS 13-14 with blunt abdominal trauma following a motor vehicular accident. Diagnostic peritoneal lavage was done & showed negative results. The assessment of negative result signifies: A. No hollow viscus injury B. Absent blood in peritoneal cavity C. Needle did not reach the peritoneal cavity D. Does not rule out abdominal organ injury

208.

The most important factor in the management of contaminated wounds is A. Broad spectrum antibiotis B. Anti-tetanus prophylaxis C. Adequate debridement D. Closure without tension

209.

Whether it‟s diverticulization or exclusion, the principle behind these procedures for the doudenal injuries is to

neoplasm of the stomach Adenocarcinoma Lymphoma Leiomyosarcoma Squamous cell carcinoma

A. B. C. D.

Prevent fistula formation To rest or isolate the injure doudenum Prevent leakage Contain the inflammation at the RUQ

210.

Sphincterotomy for anal fissures is usually done at the A. Anteromidline B. Posterolateral C. Postero midline D. Anterolateral

211.

Surgical management of familial adenomatous polyposis depends on the A. Number of polyps seen B. Size of polyps C. Histologic grading D. Staging

212.

Elective colorectal cancer operation is classified as A. Clean B. Contaminated C. Clean contaminated D. Dirty

213.

Which of the following is the most important prognostic determination of survival after treatment of colorectal cancer A. Lymph node involvement B. Transmural extension C. Tumor size D. Histologic differentiation

214.

A 60 y.o. male is admitted to the hospital after passing out large amount of maroon-colored stools. At the ER, he again passed out more bloody stools as well as clots. He is pale and tachycardic, NGT aspirate is bilious. After resuscitation, which of the ff is the most appropriate initial test: A. Barium enema B. Rigid proctoscopy C. Colonoscopy D. RBC tag scan

215.

Which is the most important stimulus for triggering endocrine response to injury A. Hypovolemia B. Afferent nerve stimulation from the injured area C. Tissue acidosis D. Local wound factors

216.

Which is not true regarding gastrostomy tube feeding? A. Can be done endoscopically or open method B. Indicated for mentally obtunded patients C. Caloric requirement is based on patient‟s need D. preferably delivered in a constant flow

217.

Pellagra, dermatitis, glossitis, & peripheral paresthesias with spinal cord symptoms are due to deficiency of A. Folic acid B. Vitamin a C. Pantothenic acid D. Zinc

218.

The primary source of energy following trauma is A. Glycogen B. Lipids C. Protein D. Carbohydrates

219.

Eicosanoids are cell mediators derived from A. Glycerol B. Cholesterol C. Arachidonic acid D. Glutamine

220.

The reason why approximately one half of breast cancer are located at the upper outer quadrant is A. Its lymphatics B. The predeliction of cancers superiorly C. The larger volume of breast tissue D. Abundance of blood supply

221.

Which structures demarcates the node levels in breast cancer: A. Latissimus dorsi B. Pectoralis major C. Axillary artery D. Pectoralis minor

222.

Carcinoma of the breast among the elderly presenting as a bulky, colloid tumor is

A. B. C. D. 223.

Mucinous Medullary Adenoid Apocrine

Management of inflammatory breast carcinoma with the best response A. Neoadjuvant chemo + MRM+ irradiation B. Classical mastectomy + irradiation C. Chemotherapy+irradiation

D.

RM+hormonal manipulation

224.

Which of the following structures which when encroached by cancer gives rise to the “orange-peel” appearance of the breast? A. glands of Montgomery B. Ductal ampulla C. Lobules D. Subdermal lymphatics

225.

A 34 female patient came at the OPD with 2cm palpable mass, moveable, nontender. No axillary nodes noted. Patient has an elder sibling who died of breast ca last 2 years ago. What will be your initial diagnostic procedure? A. Needle or open biopsy B. Mammography C. Open biopsy ultrasound of the breast D. Magnetic resonance imaging

226.

Collagen synthesis is at its peak at around how many days? A. 1-2 B. 3-5 C. 5-7 D. 10-14

227.

Formation of collagen fibers by cross-linking is enhanced by lysyl oxidase together with A. Magnesium B. Copper C. Zinc D. Chromium

228.

Which of the following steroids useful in the treatment of scars and keloids A. Betamethasone B. Triamcinolone C. Prednisolone D. Cortisone

229.

Most common A. B. C. D.

230.

45 male came in to the ER with a 3 cm lacerated wound at the left leg after he was bitten by a pig. He has received tetanus toxoid and tetanus immunoglobulin 10 1/2 years ago. What would be the appropraite management at the ER? A. Toxoid + tetanus immunoglobulin+antibiotics B. Tetanus immunoglobulin only C. Debridement, leave wound open + tetanus immunoglobulin +toxoid+antibiotics D. Tt+ats+cloxacillin

231.

Superior vena A. B. C. D.

232.

The average length of the adult trachea is A. 8 cm B. 9 cm C. 10 cm D. 11 cm

233.

Squamous cell A. B. C. D.

234.

The initial diagnostic tool in assessing biliary ductal obstruction A. Alkaline phosphatase B. Ultrasonography C. Ercp D. Ct scan

235.

Progressive painless jaundice with positive “ Courvoisier‟s sign” suggest A. Choledocholithiasis B. Hydrops of gall bladder C. Pancreatic head or peri ampullary neoplasm D. Choledochal cyst

236.

The initial diagnostic procedure in patients presenting with lower GI bleed A. NGT intubation B. Proctosigmoidoscopy C. Barium enema D. Colonoscopy

237.

The main parameter for determining the clinical stage of head and neck tumors is A. Size of primary tumor B. Lymph node involvement C. Metastasis D. Functional loss

238.

Malignancy of the tongue usually occurs at the A. Tip B. Midlateral C. Central D. Posterior

239.

The second most frequent benign salivary gland neoplasm is A. Pleomorphic adenoma B. Oxyphilic adenoma

cause of hospital acquired infections are coming from the Respiratory tract Intraabdominal Urinary tract Blood

caval obstruction is commonly caused by Fibrosing mediastinitis Bronchogenic carcinoma Caval thrombosis Aortic aneurysm

ca of the lung is usually associated with Hypercalcemia Hyperkalemia Hyperglycemia Hypomagnesemia

C. D.

Papillary cystadenoma lymphomatosum Sialadenoma

240.

The acidosis in A. B. C. D.

hypovolemic shock is due to Phosphate Lactic acid Carbonic acid HCL

241.

Substance associated with increased incidence of mesothelioma A. Smoking B. Silica C. Asbestos D. Lead

242.

Prostatic carcinoma usually metastasizes to the A. Liver B. Kidneys C. Bladder D. Bone

243.

The earliest visible evidence of neoplastic transformation is A. Hyperplasia B. Metaplasia C. Dysplasia D. Angiogenesis

244.

Epstein-barr virus is associated with the following tumors, except A. Burkitts lymphoma B. Nasopharyngeal carcinoma C. Hodgkin‟s D. Hepatocellular carcinoma

245.

A normal to low CVP that does not rise with rapid administration of crystalloids indicates: A. Inadequate rate of infusion B. Improper placement of CVP C. Continuing hypovolemia D. Improper choice of crystalloids

246.

Recurrent laryngeal nerve is preserved in thyroid surgery. What is false with regards to this nerve: A. Injury to both nerves cause upper airway obstruction B. Injury to the ipsilateral nerve cause disability of phonation C. It is a branch of cranial nerve x D. 64% of the r recurrent is seen along the tracheosophageal groove

247.

Among the types of thyroid cancer, which one is the most aggressive? A. Follicular B. Papillary C. Hurthle D. Anaplastic

248.

A 45yo female patient underwent total thyroidectomy for follicular ca. 24 H after surgery, the patient developed numbness around the lips and fingers. What could have caused this? A. Transection of recurrent nerve B. Ischemia to the parathyroids C. Hematoma D. Transection of superior laryngeal nerve

249.

True regarding A. B. C. D.

250.

A 23yo male patient came in at ER with irreducible inguinal hernia of >16hours duration. Patient complained of abdominal pain with BP of 100/70, CR 120, & RR 30. The appropriate management is: A. Observe for progression of pain B. Sedate patient and reduce the hernia C. Apply ice pack over the hernia D. Schedule patient for surgery

251.

At the ER, a patient complained of renal colic. The following are the minimal laboratory evaluation you would request except: A. CBC and electrolytes B. IVP C. Bun and creatinine D. Urinalysis

252.

In the medical treatment of BPH, which among the drugs acts by reducing the intraprostatic dihydrotestosterone levels without lowering the plasma testosterone level? A. Doxazosin

hernia: Most common type of hernia in females is direct hernia Strangulation is more common in indirect than in femoral hernia Tension-free repair is best achieved with bassini technique Repair of umbilical hernia is indicated in infants with hernia > 2cm

B. C. D.

Finasteride Terazosin Tamsulosin

253.

A 48hour old baby boy was brought to the ER due to vomiting of bilious vomitus, abdominal distention, and failure to pass out meconium. What is the most likely cause? Tracheo-esophageal atresia Ileal atresia Pyloric stenosis Rectal mass

254.

A 28 kg child with ruptured appendicitis has the following laboratory results: BP=90/60; PR=110; RR=25; Na=125; Cl=95; K=3; ph=7.35; pCO2=35; pO2=100; HCO3=24. Which of the following is TRUE? A. anion gap is increased B. uncompensated metabolic acidosis is present C. actual Na deficit is 250 mEq D. the ideal intravenous fluid to infuse is D50.3%NaCl

255.

Interpret the above ABG results: A. compensated respiratory alkalosis B. uncompensated metabolic acidosis C. compensated metabolic acidosis D. uncompensated metabolic alkalosis

256.

In the above patient, which of the following is TRUE? plasma volume is 5,600 cc initial fluid resuscitation is 560 cc given as fast drip decrease in Na is primarily due to renal losses decrease in Na is dilutional in nature

257.

In the same patient, which abnormality is he liable to manifest? A. increased intracranial pressure B. dry sticky mucous membranes C. shift of oxygen-hemoglobin dissociation curve to the left D. ventricular arrhythmias

258.

Extracellular fluid deficit results in: A. increased sodium B. increased hematocrit C. decreased BUN D. decreased creatinine A 70-kg is admitted because of abdominal colic followed by vomiting of previously ingested food. Initial assessment is an Intestinal Obstruction probably from adhesions. On examination, the patient has weak and rapid pulse, depressed eyeballs, and dry tongue and mucous membranes. The most likely fluid state of this patient is: A. hypotonic dehydration B. isotonic dehydration C. hypertonic dehydration D. hypernatremia

259.

260.

TRUE statements regarding fluid and electrolyte losses in this patient: A. vomiting accounts for an insignificant amount of the volume loss B. can go into hypovolemic shock anytime C. sequestration of fluid in the urinary system accounts for majority of the losses D. compensatory fluid shift from other compartments is sufficient to maintain homeostasis

261.

Fluid replacement in this patient should begin with: A. blood C. crystalloids B. colloids D. plasma

262.

263.

264.

265.

Which of the following is an expected cellular change/s in hypovolemic shock? A. intracellular gain of Na+ and H20 B. depletion of cellular ATP C. extracellular increase of K+ D. increase in transmembrane potential A patient has a blood pressure of 70/50 mmHg and a serum lactate level of 30-mg/100 ml (normal: 6 to 16). His cardiac output is 1.9 L/min, and his central venous pressure is 2 cmH 2O. The most likely diagnosis is: A. congestive heart failure B. C. cardiac tamponade C. hypovolemic shock D. pulmonary embolus During compensation for hypovolemic shock, blood flow to which of the following area/s is under sympathetic nervous system control? A. brain B. C. small bowel C. heart D. D. liver This hemodynamic monitor is valuable to differentiate a pericardial tamponade from hypovolemia:

A. B. C. D.

CVP C. pulmonary catheter Arterial catheter D. capnograph

266.

In a multiply injured patient with acute blood loss, adequate preload to the heart is maintained initially by the: A. increase in systemic vascular resistance B. development of tachycardia C. hormonal effects of angiotensin D. hormonal effects of antidiuretic hormone

267.

An example of neurogenic shock: A. convulsion B. syncope

C. carpopedal spasm D. involuntary fine tremors

268.

Coagulation factors tested by prothrombin time (PT): A. VII C. VIII B. XII D. XI

269.

The initial hemostatic response to injury is: A. platelet formation C. vascular constriction B. hemolysis D. initiation of coagulation cascade

270.

All are major events in the hemostatic process: A. vascular dilatation C. fibrin deposition B. platelet plug formation D. hemolysis

271.

The following are suggestive of DIC: A. platelet counts of 250,000/cu.mm B. increased fibrin split products

C. normal fibrinogen level D. hypofibrinogenemia

272.

Injury to the intima of blood vessels exposes the subendothelial collagen which initiates: A. fibrinolysis C. release of cyclic AMP B. platelet aggregation D. vasoconstriction

273.

A 77-year old man is scheduled to undergo sigmoid colectomy. He denies any history of prolonged bleeding. Preoperative evaluation of hemostasis should include: A. no screening tests B. prothrombin time (PT) and partial thromboplastin time (PTT) C. platelet count, blood smear, and PTT D. platelet count, PT, and PTT

274.

The total caloric requirement of a malnourished patient is calculated using his: A. Actual weight C. 50% of ideal body weight B. 75% of ideal body weight D. Ideal body weight

275.

In prescribing the nutrition therapy, micronutrients are given at what % of RDA: A. 100 C. 60 B. 80 D. 50

276.

Nitrogen excretion is less in: A. starvation B. sepsis

C. severe burns D. elective surgery

277.

At the initial stages of septic shock it is normally expected to have a: A. normal blood volume C. cold extremities B. hypertension D. hypothermia

278.

A 38-year-old male had abdominal exploration for multiple gunshot wounds. He is febrile T 38.5 C on the 2nd postop day; HR is 98 bpm, BP 100/80. WBC count is 13000. The patient is hooked to a ventilator. The patient is developing what condition; A. SIRS C. severe sepsis B. sepsis D. septic shock

279.

On the 5th day post-op the patient in addition to the above findings now shows erythema and draining pus from the abdominal incision site. The patient now is developing; A. SIRS C. severe sepsis B. sepsis D. septic shock

280.

On the 10th day post-op, the patient is persistently febrile with increase in WBC count (18000) and this time with oliguria that is unresponsive to fluid resuscitation. The patient now is developing; A. SIRS C. severe sepsis B. sepsis D. septic shock

281.

The above patient was given pressor agents to improve his condition, despite this he remains oliguric. The patient now has: A. SIRS C. severe sepsis B. sepsis D. septic shock

282.

Immediately after mastectomy your patient asks if any of the following promotes wound healing, which one wills you advise your patient? A. Vitamin C C. Vitamin B B. Zinc supplementation D. carbohydrate rich foods

283.

Severe cases of hidradenitis suppurativa in the groin area are best managed by excision of the involved area and: A. closure by secondary intention B. delayed primary closure C. primary closure D. partial thickness skin grafts

284.

An elderly cancer patient fell and sustained a deep lacerated wound over the right forehead about 9 cm length. If wound infection later develops in this patient. The major cause of impaired wound healing is: A. anemia C. DM B. local wound infection D. all of the above

285.

What technique of wound closure is recommended in a patient with ruptured appendix with spreading peritonitis? A. primary C. secondary B. delayed primary D. tertiary

286.

If the lacerated wound is sutured closed, this healing is known as; A. primary C. tertiary B. secondary D. delayed primary

287.

Wound healing in this patient may be impaired because of his; A. age C. type of wound B. depth of the wound D. cancer

288.

Infection in the above wound is partly controlled by the presence of what cells; A. macrophages C. endothelial cells B. fibroblast D. T lymphocytes

289.

Important activities of macrophages during wound healing; A. wound debridement C. activation of coagulation cascade B. arginine synthesis D. fibrin clot

290.

The ABC‟s of resuscitation includes the following: A. establish adequate airway B. bleeding should be controlled by applying tourniquet C. circulation should be maintained by rapid infusion of blood D. oxygen should be delivered if necessary

291.

A 25-year-old male is brought to the ER after he sustained a stab wound to the 5th intercostals space. PE: conscious, stretcher-borne, vitals are as follows: blood pressure 90/50, pulse 110/min and respiratory rate 30/min. Which of the following statements is true: A. primary survey and initial resuscitation should be instituted one after the other B. a complete detailed physical exam should be performed so as not to miss any injury C. large-bore IV lines should be inserted D. blood for CBC and x-matching should be drawn

292.

An 8-year old boy has a closed angulated deformity of his right forearm. VS: BP = 99 mm Hg systolic and PR = 110 Identify the most emergent problem of the victim. Write any of the following: A. Airway or Cervical Spine C. Circulation B. Breathing D. Disability or Neurologic Injury

293.

A 7-year old boy is gasping for breath. He can speak clearly, but complains of chest pain and an inability to get his breath. He is becoming increasingly anxious. VS: BP = 100 mm Hg systolic and PR = 110 Identify the most emergent problem of the victim. Write any of the following: A. Airway or Cervical Spine C. Circulation B. Breathing D. Disability or Neurologic Injury

294.

A 25-year-old man is unconscious, withdraws from painful stimuli, eyes are closed, but they open in response to pain, and he is making unintelligible sounds. His pupils are equal, and both react sluggishly to light. His Glasgow coma scale score is: A. 5 C. 11 B. 8 D. 14

295.

After control of this patient's airway is achieved, the first diagnostic study to obtain is: A. x-rays of the skull C. x-rays of the cervical spine B. CT scan of the head D. carotid angiography

296.

A 19-year-old woman presents with a non-bleeding stab wound, 1 cm long, in the anterior triangle of her neck, 3 cm above her left clavicle. Examination of the neck reveals that the wound has penetrated the platysma muscle, and that the left carotid pulse is normal; there is no hematoma or bruit. Appropriate management of this patient: A. immediate tracheostomy B. nasogastric intubation C. chest x-rays D. formal neck exploration under general anesthesia

297.

Following an automobile accident, a 30-year-old woman is discovered to have a posterior pelvic fracture. Hypotension and tachycardia respond marginally to volume replacement. Once it is evident that her major problem is free intraperitoneal bleeding and a pelvic hematoma in association with the fracture, appropriate management would be: A. application of antishock trousers and exploratory laparotomy B. arterial embolization of the pelvic vessels C. exploratory laparotomy and possible bilateral internal iliac artery ligation D. external pelvic fixation to stabilize the pelvis

298.

Large amount of free intraperitoneal blood is seen in a patient with penetrating abdominal injury after an abdominal ultrasound, the next best step is to: A. apply antishock trousers and exploratory laparotomy B. arterial embolization of the pelvic vessels C. exploratory laparotomy D. external pelvic fixation to stabilize the pelvis

299.

A 6-year-old is having episodes of watery diarrhea for the past three days. She is lethargic, irritable and restless. The heartbeat is rapid but the pulses are weak. The child should: A. stay home and call the pediatrician for advice B. stay home and have a cold bath C. proceed to the nearest hospital emergency room D. get a glass of cold fruit-juice to drink

300.

Over-enthusiastic fluid resuscitation should be avoided in this child because of: A. potential liver failure B. acute myocardial infarction C. venous thrombosis and rupture D. potential cerebral injury

301.

A study on ruptured appendicitis will be conducted in a tertiary university hospital that includes all age groups and all genders on the rate of SSI between primary vs. delayed primary closure. What is the possible confounder in this study? A. all age groups B. ruptured appendicitis C. SSI D. primary vs. delayed primary

302.

The most common type of inguinal hernia in both males and females is: A. Indirect B. Direct C. Femoral D. Mixed.

303.

The Cooper‟s ligament repair for direct or femoral hernia is popularized by: A. Bassini B. McVay C. Halsted D. Ferguson

304.

A 22 year old female consulted a physician because of a 2x2x2 cms, movable, well circumscribed, non tender mass, noted accidentally while taking a bath. Physical examination was consistent with the findings above. Most likely diagnosis is: A. Breast Ca B. Fibrocystic change C. Fibroadenoma D. Phylloides tumor

305.

The next thing to do on the above patient is: A. Reassurance and observation B. Ask for a mammogram C. Partial mastectomy D. Hormonal treatment

306.

Ivory an 18 year old female consulted because of breast tenderness becoming more severe just before menses. PE – showed asymmetric nodularities on the upper outer quadrant of both breasts with no dominant mass . Menstrual cycle is monthly and regular. Most likely diagnosis is: A. Sclerosing adenosis C. Fibrocystic change B. Normal premenstrual tension D. Breast Ca

307.

Sarah Jane a 35 year old female consulted because of an intermittent bloody nipple discharge on the right breast PPE- no mass noted on the right breast Most likely diagnosis is: A. Papillary cancer C. Bleeding galactocoele B. Mammary duct ectasia D. Intraductral papilloma

308.

Chemotherapy in breast carcinoma is:

A. B. C. D.

given to those with at least 4 positive axillary metastasis usually a combination of multiple agents free of any side effects used to control local recurrence

309.

Breast malignancy that is known to have the highest incidence of bilaterality and multicentricity: A Paget‟s disease of the breast C. Papillary carcinoma B. Lobular carcinoma D. Medullary carcinoma

310.

The next appropriate thing to do in managing the above case will be: A. Local wound exploration B. CT Scan C. Immediate surgery D. Triple angiography of the abdomen

311.

Fifteen minutes later the BP became 60 palpatory, PR-120/min accompanied by distention. You will recommend: A. CT scan C. MRI B. proceed with local wound exploration D. Immediate surgery

312.

The most commonly injured intraabdominal organ following a blunt abdominal trauma; A. Pancreas B. spleen C. small intestine D. colon

313.

A 55 y/o male, smoker underwent FNAB for a 4cm preauricular mass. Results showed (+) for malignant cells .The patient is most probably suffering from: A. Pleomorphic adenoma C. mucoepidermoid carcinoma B. Follicular carcinoma D. papillary carcinoma

314.

The appropriate treatment for a benign parotid neoplasm is: A. “shelling out” of the tumor B. superficial parotidectomy

abdominal

C. total parotidectomy D. radiation therapy

315.

Most common histology of head-neck cancers is: A. Adenocarcinoma C. squamous cell carcinoma B. papillary carcinoma D. Adenosquamous carcinoma

316.

The most common causative agent being implicated in head-neck cancers is: A. UV radiation B. Tobacco C. nitrates D. alcohol

317.

The site of injury corresponds to which zone of the neck? A. Zone I C. Zone III B. Zone II D. Zone IV

A 60 year old woman with chronic renal failure and undergoing hemodialysis 2-3x/week develops tumoral calcinosis. parathyroid hormone is markedly elevated while her Ionized Calcium is normal.

Her

318.

The diagnosis of the above is: A. Primary Hyperparathyroidism B. Secondary Hyperparathyroidism C. Tertiary Hyperparathyroidism D. Parathyroid carcinoma.

319.

The course of papillary thyroid cancer is best described by which of the following statements?: A. Metastases are rare; local growth is rapid B. Local invasion and metastases almost never occur C. Metastases frequently occur to cervical lymph nodes, distant metastases and local invasion are rare D. Rapid widespread metastatic involvement of the liver ,lungs and bone results in a 10% 5 yr survival rate

320.

The most important prognosticating factor in thyroid cancer is: A. tumor size C. lymph node metastases B. patient‟s age D. histologic grade

321.

43 years old female present with diffuse enlargement of the thyroid gland accompanied by palpitation, heat intolerance and weight loss. She is most probably suffering from : A. Follicular Cancer C. Grave‟s disease B. Plummer Vinson‟s disease D.Colloid goiter

322.

A 50 years old male has just undergone a total thyroidectomy for follicular cancer of the thyroid. On the third hour post-op, he experienced circumoral numbness. He is probably suffering from A. Frey‟s syndrome C. hypocalcemia B. hypothyroidism D. hypokalemic acidosis

A 16 year old high school student came to you with a complaint that her friends took notice of an anterior neck enlargement since she started on her weight reduction program. You were indecisive as to whether there really was a thyromegaly. The rest of the P.E. were normal 323.

Which would be most helpful in determining whether there really is a thyromegaly or none: A. a second opinion from an endocrinologist B. MRI of the neck

C. Thyroid Scanning D. serum TSH, T3 & T4 324.

A 72-year old male consults because of change in bowel habits for the last 2 weeks. On rectal exam, a hard, irregular mass was noted at 6 cm from the anal verge The most likely diagnosis here is: A. Rectal carcinoid C. Rectal Polyp B. Rectal carcinoma D. Anal canal carcinoma

325.

The most appropriate initial diagnostic procedure to be done is: A. Ultrasound of the abdomen C. Chest x-ray B. Proctosigmoidoscopy D. CT scan of the pelvis

326.

Majority of peptic ulcer patients are successfully managed medically or conservatively. Which of the following complications is the most common indication for surgical intervention? A. Perforation C. Gastric outlet obstruction B. Massive GI bleeding D. Malignant degeneration

327.

A 48 year old company manager who has been taking proton pump inhibitor irregularly for the past 2 years because of on & off epigastric discomfort presents in the emergency room with hematemesis and melena. This patient should be: A. Aggressively resuscitated with fluids and measures to localize site of bleeding should be done in preparation for possible surgery B. Explored immediately because bleeding is an indication for surgery C. Give massive doses of anti-ulcer medications D. Watchful waiting for 24-48 hours.

328.

A 43 year old male patient with on & off epigastric pain during the past 15 months complains of sudden severe epigastric pain later on becoming generalized. An upright chest file taken at the ER shows a strip of lucency underneath the right hemidiaphragm. This patient has: A. Penetrating peptic ulcer B. Acute pancreatitis C. Perforated diverticulitis D. Perforated peptic ulcer.

329.

Above patient is best managed by: A. Aggressive medical treatment of PUD B. Laparotomy after resuscitation / IV antibiotics C. Fluid resuscitation, nasogastric decompression and somatostatin D. Emergency ERCP

330.

A 30 y/o male who usually has dripping fresh blood after defecation suddenly had anal pain and a smooth, tender, 1 cm diameter rounded mass coming out of the anus. This is most likely: A. peri-anal abscess B. internal hemorrhoids 3rd degree C. Anal wart D. thrombosed internal hemorrhoids

331.

A 65 y/o female was referred for surgical consult because of abdominal pain and distention. Findings in the scout film of the abdomen that will be highly suggestive of intestinal obstruction: A. Air in the distal rectum B. Pneumatosis intestinales C. luscency beneath the right hemidiapragm D. step ladder abnormality

332.

A 75 y/o female is being worked up for paralytic ileus The electrolyte that is usually A. Potassium C. calcium B. Sodium D. Magnesium

333.

A significant and independent influence on the prognosis of a patient who had an esophageal resection for cancer is: A. Tumor penetration of the esophageal wall B. Cell type C. Degree of cellular differentiation D. Location of the tumor in the esophagus

334.

Hepatic resection should be considered for localized metastatic spread from which of the following primary site? A. colorectal C. Lungs B. stomach D. Breast

335.

The standard of treatment for patient with biliary colic and cholecystolthiasis by ultrasound is A. Cholecystectomy C. Stone Dissolution B. Cholecystostomy D. Endoscopic Stone removal

336.

In an infected obstructed common bile duct producing cholangitis which of the following is the best option for drainage in a very poor risk patient ? A. cholecystostomy

below normal is:

B. operative CBD exploration and T tube choledochostomy C. ERCP and stenting D. MRICP 337.

CT scan finding that is highly indicative of infected pancreatic necrosis; A. edema C. calcifications B. dilated pancreatic duct D. gas bubbles in the retroperitoneum

338.

When gallstone ileus occurs , obstruction is most frequent in : A. duodenum C. distal ileum B. jejunum D. Sigmoid colon

339.

The most common cause of Acute Cholecystitis is : A. cystic duct obstruction B. E. coli infection C. Gall bladder polyp near the neck D. multiple gallstones.

340.

The strongest evidence of presence of gallstones by ultrasound is : A. Edema and thickening of gallbladder wall B. Contracted gallbladder C. Dilated extrahepatic bile ducts D. High level echoes with posterior acoustic shadowing.

A 45 year old jaundiced patient who has been diagnosed to have gallbladder and common bile duct stones by ultrasound 6 months ago is brought to the ER because of abdominal pain , chills, high grade fever and hypotension. PE: icteric sclerae , abdomen slightly tender with guarding RUQ. 341.

The patient most likely is having which of the following: A. acute cholecystitis C. Acute suppurative obstructive cholangitis B. empyema of the gallbladder D. ruptured gallbladder

342.

The problem of patient in the previous number is best managed by: A. administration of oral antibiotics B. careful observation and follow up C. surgical/endoscopic decompression of biliary tree after resuscitation and antibiotics D. give systemic antibiotics and steroids.

343.

The most likely diagnosis in a 70 y/o male presenting with significant weight loss accompanied by progressive jaundice, anorexia, pruritus and tea colored urine. PPE showed a palpable non tender gallbladder? A. hepatoma C. pancreatic head cancer B. gallbladder cancer D. gallbladder empyema

344.

The severity of pancreatitis, particularly in those patient who is not improving after 24 hours of medical management can be assessed using: A. Angiography C. Dynamic CT Scan B. Scout film of the abdomen D. MRICP

345.

Which of the following clinical clues is a reliable symptoms of arteriosclerosis A. Poor hair growth C. Cold hands and feet B. Deformed toenails D. Intermittent claudication.

346.

What is the Most Common symptom of acute aortic dissection? A. Syncope C. Hemiparesis B. Severe pain D. Congestive heart failure.

347.

This is the BEST procedure for a patient with acute head injury secondary to a vehicular accident: A. CT scan C. Plain skull x-rays B. Angiography D. Electroencephalogram

348.

Abrasion wounds on the face are best treated with: A. wound cleansing and debridement with normal saline B. split thickness skin grafting C. laser surgery D. all of the above are correct

349.

Essential elements in the survival of a skin graft: A. Antibiotics C. Vascularity B. Dry dressing D. Nerve supply.

350.

Most common cause of UTI in children: A. E. coli B. Staph saprophyticus

351.

obliterans?

C. Klebsiella, Enterobacter D. Proteus sp

A massively bleeding posterior duodenal ulcer, which of the following is most likely involved: A. Left gastric artery B. Gastroduodenal artery

C. D.

Short gastric artery Left gastroepiploic artery

352.

Which of the following would inhibit parietal cell acid secretions? A. Histamine B. Gastrin C. Acetylcholine D. Prostaglandins

353.

The most common symptom associated with a Meckel‟s diverticulum is: A. Crampy abdominal pain B. Periumbilical pain C. Blood per rectum D. Watery diarrhea

354.

The typical visual field deficit caused by a pituitary adenoma with suprasellar extension is: A. Bitemporal hemianopsia B. Homonymous hemianopsia C. Superior quadrantanopsia D. Inferior quadrantanopsia

355.

The evaluation of a comatose patient with a head injury begins with: A. Evaluation of cardiovascular system B. Evaluation of pupillary reflexes C. Establishment of an airway D. Plain film of the skull

356.

The radial nerve is at greatest risk for injury with which fracture: A. Fracture of the surgical neck of the humerus B. Fracture of the shaft of the humerus C. Supracondylar fracture of the humerus D. Olecranon fracture

357.

An early A. B. C. D.

358.

The most common site of involvement for skeletal tuberculosis is the: A. Tibia B. Femur C. Pelvis D. Spine

359.

A 65-year old woman with a history of chronic constipation is transferred from a nursing home because of abdominal pain and marked abdominal distention. On examination, her abdomen is found to be distended and tender in the left lower quadrant. The most likely diagnosis is: A. Appendicitis B. Carcinoma of the colon C. Volvulus of the sigmoid colon D. Small bowel obstruction

360.

A 68-year old man presents with crampy abdominal pain and distention vomiting. Findings on PE are positive for healed abdominal scars. X-ray reveals multiple gas fluid levels. The WBC count is 12,000. The most likely diagnosis is: A. Small bowel obstruction due to adhesions B. A hernia C. Appendicitis D. Gallstones and ascites

361.

A 56-year old man has suffered from intermittent claudication for 5 years. He has recently developed cramping abdominal pain that is made worse by eating. He has a history of a 15-lb weight loss. The most likely diagnosis is: A. Chronic cholecystitis B. Chronic intestinal ischemia C. Peptic ulcer D. Abdominal aortic aneurysm

sign of compartment syndrome in the hand includes: Pain with passive stretch at the digits Absent radial pulse Motor paralysis Swelling of the digits

A 28-year old man with a history of emotional disturbance enters the hospital with a history of weight loss and regurgitation of food. Regurgitation is worse when he lies down. The most likely diagnosis is: A. Hiatal hernia B. Cancer of the esophagus C. Duodenal ulcer D. Achalasia A 38-year old man with a history of fever associated with abdominal pain of several weeks duration presents now with a sudden onset of explosive abdominal pain and vomiting. Flat plate x-rays reveals air under the diaphragm. CT scan shows mesenteric lymphadenopathy and splenomegaly. The most likely diagnosis is: A. Tuberculosis enteritis B. Typhoid enteritis

C. D.

Primary peritonitis Ulcerative colitis

Splenectomy is often A. B. C. D.

indicated in the management of: Hereditary spherocytosis Hereditary neurofibromatosis Aplastic anemia Hashimoto‟s disease

A painless distended A. B. C. D.

gallbladder palpable on PE of a jaundiced patient is strongly suggestive of: Empyema of the gallbladder Mirizzi‟s syndrome Gallstone impacted in the ampulla Pancreatic carcinoma

Which of the following suggest unresectability of a left upper lobe lung cancer: A. Hemoptysis B. Malignant pleural effusion C. A cough specimen with positive sputum cytology D. Clubbing and blueness of fingers Which of the following studies is contraindicated in a drowsy patient with papilledema whom one suspects of having acute closed head trauma: A. Carotid arteriography B. Lumbar puncture C. CT scan D. Echoencephalography The appropriate antibiotic for a patient with a cellulitis of a leg due to streptococcus is: A. Penicillin B. Erythromycin C. 3rd generation cephalosphorins D. Cloxacillin Massive bleeding from the lower GI tract (beyond the ligament of Treitz) is most often due to: A. Diverticulosis B. Meckel‟s diverticulum C. diverticulitis D. colonic carcinoma A 54-year old male has been complaining of recurrent hypogastric pain for almost 6 months. For almost 3 weeks prior to consult, he had noticed that he is passing air during micturition. The most common cause of the above condition is: A. carcinoma of the bladder B. tuberculosis of the urinary tract C. carcinoma of the sigmoid colon D. diverticulitis of the colon A 42-year old woman has been complaining of intermittent gross hematuria. On urinalysis, she has significant pyuria but without bacilluria. The condition is most commonly due to: A. urethritis B. bladder stone C. perinephric abscess D. tuberculosis of the kidney A 25-year old male had episodes of severe vomiting due to a gastric outlet obstruction. He appears pale and dehydrated. The likely metabolic abnormality the patient would have is: A. hypochloremic acidosis B. hyponatremic acidosis C. hypochloremic alkalosis D. hypernatremic alkalosis A 1week old infant presents with moderate respiratory distress and tympany in only one hemithorax. The most likely diagnosis is: A. spontaneous pneumothorax B. eventration of the diaphragm C. Bochdalek hernia D. Atelectasis A little boy aspirated lodged in the: A. B. C. D.

a peanut. He is cyanotic. On expiration, his mediastinum shifts to the right. The peanut most likely is

A 40-year old man is injury: A. B. C. D.

hit by a car and sustains an injury to the pelvis. Which of the following is most indicative of a urethral

right main stem bronchus left main stem bronchus trachea esophagus

hematuria high-riding prostate on rectal examination oliguria scrotal ecchymosis

A 64-year old man has mild upper abdominal pain. On contrast CT-scan, a 5 cm lesion in the left lobe of the liver enhances and then decreases over a 10-minute period from without to within. The most likely lesion is a: A. hepatic adenoma B. amoebic abscess C. fatty infiltration D. carvernous hemangioma A 28-year old male has had a nonseminomatous testicular cancer treated. In following this patient for possible recurrent tumor, the most useful serum marker would be: A. carcinoembryonic antigen B. alpha fetoprotein C. prostate specific antigen D. alkaline phosphatase The most common cause of esophageal rupture or perforation: A. Endoscopic injury B. Blunt chest trauma C. Boarhaave‟s syndrome D. Carcinoma The superior mesenteric artery communicates with the celiac artery via the: A. Pancreatico-duodenal artery B. Splenic artery C. Hepatic artery D. Dorsal pancreatic artery The most common offending organism in pyogenic osteomyelitis is: A. Escherichia coli B. Staphylococcus aereus C. Pseudomonas aeruginosa D. Entococcus In patients who developed a documented episode of deep vein thrombosis, the most frequent and significant long term sequela is: Claudication Recurrent foot infection Development of stasis ulcer Pulmonary embolism In an arterial stenosis, the most critical factor is: A. The length of the stenosis B. The cross-sectional area of the stenosis C. The length and area are equally important D. Whether the stenosis is smooth or has an irregular countour A 60-yr old man with a history of atrial fibrillation is found to have a cyanotic cold right lower extremity. The embolus most probably originated from: A. An atherosclerotic plaque B. Deep vein thrombosis C. Lungs D. Heart Which of the following elements is not a component of venous thrombogenesis: A. Incompetent valves in perforating vein B. Disruption of endothelial intima C. Venostasis D. Hypercoagulability In a patient suffering from an acute arterial embolic phenomenon, if the ischemia is not relieved, which of these following tissues is the first to be irreversibly damaged: A. muscle B. nerve C. fat D. synovial membrane Which of the following clinical manifestation may suggest tentorial herniation: A. contralateral mydriasis B. decorticate rigidity C. contralateral hemiparesis D. ipsilateral hemianopsia The most common complication of lung abscess is: A. Empyema B. Pneumothorax C. bronchopleural fistula D. osteomyelitis of the ribs The most common posterior-superior sulcus chest tumor in a 6-month old child is: A. Neuroblastoma B. Teratoma

C. D.

cystic hygroma ganglioneuroma

The most common cause of spontaneous pneumothorax is: A. tuberculosis B. emphysematous blebs C. pneumonia D. bronchial rupture What substance is released by macrophages in order to activate T helper/inducer (CD4+) lymphocytes: A. Interleukin 1 B. Interleukin 2 C. Interleukin 3 D. Interleukin 4 A 27-year old hypertensive, diabetic woman is admitted for cadaveric renal transplantation. She is blood type B and has had four transfusions of packed cells over the preceding 6 months. Which of the following factors would preclude transplantation: A. Donor blood type O B. Positive crossmatch C. Two-antigen HLA match with donor D. Blood pressure of 180/100 mm Hg The primary mechanism of action of cyclosporine A is inhibition of: A. Macrophage function B. Antibody production C. Cytotoxix T-cell effectiveness D. Interleukin 2 production Which of the following malignancy exhibits a slow growing, late metastasizing characteristic: A. Squamous cell carcinoma B. Basal cell carcinoma C. Melanoma D. Basosquamous carcinoma Following the Goodsall-Salmon‟s rule in fistula in ano, an anteriorly located opening less than 3 cm from the anal opening follows: A. Straight tract B. Curve tract C. No tract D. S-shaped tract One of the following statements is correct regarding hyponatremia: A. Serum sodium determination would give a normal result B. Hypoglycemia may be a cause C. Management could either be water restriction or replacement depending on the cause D. Hyperactive deep tendon reflex is a manifestation A 60 kg lean male would normally have this amount of body water: A. 36 liters B. 24 liters C. 30 liters D. 48 liters A practical but useful clinical parameter that can be used to assess the adequacy of volume replacement in patients suffering from hypovolemia is: A. Central venous pressure measurement B. Urine output C. Blood pressure D. Pulse rate A 42-year old man who has been on prolonged total parenteral nutrition (TPN) administration was noted to have scaly, hyperpigmented lesions over the acral surface of elbows and knees and alopecia. The most likely cause of the condition is: A. Essential fatty acid deficiency B. Excess glucose calories C. Hypomagnesemia D. Zinc deficiency A 40-year old man is found to have severe metabolic acidosis with a high anion gap. The most likely cause is: A. Diarrhea B. Renal tubular acidosis C. Ureterosigmoidostomy D. Methanol ingestion

50-57

A 60-year old female, post mastectomy for breast cancer, presents with headache backache and frequent vomiting. She is extremely thirsty and stuporous. The test most likely to identify the cause is: A. Serum calcium determination B. Serum sodium determination C. Serum potassium determination

D.

Serum glucose determination

The Trendelenburg‟s (head down) position is beneficial in the initial management of which type of shock: A. Hypovolemic shock B. Cardiogenic shock C. Septic shock D. Neurogenic shock The most common form of burn is: A. scald burns B. flame burns C. chemical burn D. electrical burn A 20 year old man has lymphadenopathy behind and inferior to his right ear. Biopsy shows the lesion to be a lymphosarcoma. The most likely site of the primary tumor is: A. Floor of the mouth B. Nasopharynx C. Buccal mucosa D. Mediastinum The most frequent organism in highest density that can be isolated from contamination following colon perforation are: A. Gram positive aerobes B. Gram negative aerobes C. Gram positive anaerobes D. Gram negative anaerobes The most clinically significant early physiologic abnormality in post traumatic pulmonary insufficiency is: A. Increased physiologic shunt B. Increased dead space C. Cardiogenic pulmonary edema D. Pulmonary fibrosis 354. A. B. C. D.

Overall bone mass increases up to what age afterwhich there is an overall decrease in bone mass: 10 to 15 years of age 15 to 20 years of age Answer: D; Schwartz 8 th ed p.1654 20 to 25 years of age 30 to 35 years of age

A. B. C. D.

After multiple myeloma, the most common primary malignant bone tumor is: Osteosarcoma Ewing‟s sarcoma Answer: A; Schwartz 8th ed p.1661 Chodrosarcoma Fibrosarcoma

355.

356. A. B. C. D. 357. A. B. C. D. 358. A. B. C. D. 359.

The best treatment for septic arthritis of the hip is: Repeated daily aspiration Arthroscopic drainage Surgical drainage/Arthrotomy with antibiotic therapy Antibiotic therapy only Answer: C; Schwartz 8th ed p1678 The most common form of inflammatory arthritis: Osteoarthritis Rheumatoid Arthritis Septic Arthritis Psoariatic Arthritis Answer: B; Schwartz 8 th ed p.1679 The gold standard of treatment of femoral shaft fractures in adults is: Closed reduction and hip spica application Open reduction and reamed intramedullary nailing Closed locked antegrade intramedullary nailing Plating Answer: C; Schwartz 8th ed p.1688

Initial treatment of Talipesequinovarus (clubfoot) is: A. Serial Casting immediately after birth/time of diagnosis B. Immediate operative treatment C. Dennis Brown Splint D. Pavlik harness Answer: A; Schwartz 8th ed p.1718

360. A. B. C. D.

A unique feature of immature bone is the capacity to undergo plastic deformation without breaking and to sustain an incomplete fracture also called: Nighstick fracture Monteggia fracture Galleazzi fracture Greenstick fracture Answer: D, Schwartz 8th ed p.1698

361.

Inflammation of the tendons in the first dorsal compartment containing the Abductor pollicis brevis and extensor pollicis brevis tendons is called: A. Carpal Tunnel Syndrome B. Trigger finger C. De Quervain‟s Tenosynovitis D. Lateral Epicondylitis Answer: C; Schwartz 8th ed p.1763

11.In doing an otoscopic examination of an adult , the ear is pulled a. upward and backwards b. downward and anteriorly c. upward and forward d. downward and posteriorly ans : a page 5 Boise funadamental of otolaryngology 12.This is a clinical hearing test using tunning fork , that compares the bone conduction of the examinaer from that of the patient. a. Rinne Test b. Weber Test c. Swabach Test d. All of the above Ans. c page 9 Boise funadamental of otolaryngology 13. This nerve controls the muscle of facial expression which is easily observe during face and neck examination a. facial nerve ( CN VII) b. vagus nerve ( CN X) c. trigeminal nerve ( CN V) d. oculomotor nerve ( CN III) Ans. A page 21 Boise funadamental of otolaryngology 14.The nasolacrimal duct empties into the: a. anterior ethmoids b. inferior meatus c. lacrimal gland d. sphenoethmoidal recess Ans. B page 179 Boise funadamental of otolaryngology 15. A 3 year old boy was brought to the ER due to recurrent epistaxis. On flexible nasal endoscopy, there was a bulging mass at the nasopharyngeal area. What is the \ possible diagnosis of the patient a. Juvenile angiofibroma b. Inverted papilloma c. Nasal polyp d. Fibrous dysplasia Ans. A page 245 Boise funadamental of otolaryngology 16.This is a autosomal recessive trait characterized by situs invesus, bronchiectasis, sinusitis: a. cystic fibrosis b. kartagener syndrome c. Albrights syndrome d. Pierre robin syndrome Ans. B page 269 Boise funadamental of otolaryngology 17. This is a cellulitis or phlegmonous inflammation of the superior compartment of the suprahyoid space . The most common cause is odontogenic in origin a. Vincents Angina b. Ludwigs angina c. Trench mouth d. Retropharyngeal abscess Ans. B page 359 Boise funadamental of otolaryngology 19.the cricothyroid muscle is innervated by : a. recurrent laryngeal mnerve b. external br. of superior laryngeal nerve c. internal branch of superior laryngeal nerve d. vagus nerve Ans B page 387 Boise funadamental of otolaryngology 20.The most common benign gland tumor of children of parotid gland is: a. lymphangioma b. hemangioma c. pleomorphic adenoma d. mucoepidermoid CA ANS B page 325 Boise funadamental of otolaryngology 21.A 22 year old construction worker comes for consult because an unknown liquid got into his eyes. The very first thing to do is: a) instill antiseptics b) do copious water irrigation

c) d)

assess vision put topical antibiotics

23. A 65 year old filipino male comes in for bilateral insidious painless progressive visual deterioration. The most likely diagnosis would be: a) cataracts b) age related macular degeneration c) central retinal artery occlusion d) congestive angle closure glaucoma 24.A 55 year old male known diabetic with moderately severe non proliferative retinopathy comes in for 6 months blurring of vision not corrected with pinhole. The most likely diagnosis is: a) macular edema b) traction retinal detachment c) incipient cataract d) vitreous hemorrhage 25.A 41 year old female complains of slightly congested eyes with headache after prolonged near work, relieved by rest. She may need a) concave lenses b) convex lenses c) intraocular lenses d) binocular lenses 26.A patient complains of poor vision at distance, good vision at near. She may need a) convex lenses b) cylindrical lenses c) concave lenses d) binocular lenses 30.A chronic granulomatous inflammation of the meibomian gland is a a) cyst b) stye c) chalazion d) dermoid 1.

Following Goodsall‟s rule, a fistula in-ano with an external opening located 4 cm from the anal verge would have its internal opening located : A. B. C. D.

in the anterior midline in the posterior midline radially, in the same quadrant radially, in a separate quadrant

2. In patients with rectal cancer, Computed Tomography scanning best evaluates: A. The B. The C. The D. The

presence or absence of hepatic metastases. depth of rectal wall penetration. presence of lymph node metastases. configuration of the primary tumor

1.Ultrasonography incidentally revealed a single 1 cm gallstone in an asymptomatic 55 year old. Which of the following is the recommended treatment? A. B. C. D.

Observation and follow-up Laparoscopic cholecystectomy Open cholecystectomy Bile acid dissolution therapy

2. 70-90% of primary gastrinomas are located in the area defined by a triangle with points located at the junction of the cystic duct and common bile duct, the second and third portion of the duodenum, and the neck and body of the pancreas. This area is known as the Triangle of A. Calot B. Hesselbach C. Passaro D. Bermuda 1. Histologic examination of a thyroid gland tumor reveals psammoma bodies. This finding indicates that the lesion is: A. Follicular Carcinoma B. Medullary Carcinoma C. Papillary Carcinoma D. Hurtle Cell Carcinoma 2.

A 40 year old female was diagnosed with Papillary Thyroid Carcinoma. She presents with a hard thyroid mass, cervical lymphadenopathies, and several coin lesions on chest x-ray. Clinical staging for this patient would be: A. Stage I

B. Stage II C. Stage III E. Stage IV 3. A 45 year old woman has a mobile lump in the upper outer quadrant of her right breast. An open excisional biopsy reveals a 1.5 cm lobular carcinoma in situ with negative margins. With regards to further management, which of the following statements is true? A. B. C. D.

Careful clinical follow-up without further surgical intervention is sufficient at this time. Mirror image biopsy of the contralateral breast is warranted. Total mastectomy, without axillary dissection, is indicated. Lumpectomy with radiation therapy is the recommended treatment.

1. During the initial assessment of the multiply injured patient, the main goal and objective of the Primary Survey Is to: A. B. C. D.

obtain a detailed history and complete physical examination stabilize the patient and comfort the relatives detect and treat immediately life-threatening injuries detect and treat immediately limb-threatening injuries

2. In performing maneuvers to ensure airway patency, it is most critical to: A. B. C. D.

provide 100% oxygen have intravenous lines running stabilize the cervical spine have intubation set ready

1. Respiratory distress in cases of congenital diaphragmatic hernia is a result of A. B. C. D. Answer

inability of the ipsilateral lung to expand due to the herniated abdominal viscera recurrent aspiration pneumonia due to concomitant gastroesophageal reflux splinting of the contralateral diaphragm from distention of the intestinal tract incomplete development of the lung resulting in inadequate area for gas exchange

#

questions

page

B

1

Patients with either external or middle ear lesions usually have: A. Conductive hearing loss B. Sensorineural hearing loss C. Mixed hearing loss D. none of the above MPL: 0.75

46

D

2

Oral or systemic antibiotics are indicated in Otitis Externa when: A. One is not sure if the patient has Otitis Externa or Otitis Media B. The patient complains of severe pain C. Debris is seen within the external auditory canal D. The infection extends beyond the limits of the canal MPL: 0.75

82

B

3

The most common etiology of acute otitis media in all age group A. Staphylococcus sp. B. Streptococcus C. H. Influenzae D. Moraxella catarrhalis MPL : 1.00

99

B

5

Which layer is missing when a tympanic membrane perforation heals A. Epidermal layer B. Fibrous layer C. Mucosal layer D. None MPL: 0.75

D

6

The most common intracranial complication from supurative Otitis Media is: A. epidural abscess B. subdural abscess C. brain abscess D. meningitis MPL: 0.75

C

10

Most accurate test in detecting deafness in neonates and in children A. Play audiometry B. Pure tone audiometry C. Auditory Brainstem Evoked Response D. Otoacoustic emission MPL: 0.50

91

118

67

C

11

The most common complaint of patients with otomycosis: A. Earache B. Tragal tenderness C. Itchy ear D. Non mucoid ear discharge MPL: 1.00

82

B

12

Sudden hearing loss with concurrent dizziness while scuba diving suggests: A. Acoustic neuroma B. Barotrauma C. Meniere‟s disease D. Benign Positional Paroxysmal Vertigo MPL: 0.75

94

14

Conditions predisposing to the development of otitis media A. Upper respiratory tract infection B. Allergic rhinitis C. Breastfeeding D. Cleft palate MPL: 0.75

A

21

A 3 year old boy was refered to the ENT service due to edema of the eyelids. 236-264 Pertinent physical examination: normal visual acuity, (-) chemosis, (-) proptosis, full extraocular muscles, (+) profuse purulent nasal discharge. What is your assessment? A. Periorbital cellulitis B. Orbital cellulitis C. Subperiosteal abscess D. Orbital abscess MPL: 0.75

B

22

The most common cause of epistaxis in the elderly A. Trauma B. Atherosclerosis and hypertension C. Foreign body D. Blood dyscrasia MPL: 1.00

236

C

27

202

A

28

Which of the following best describe a patient with allergic rhinitis? A. Presence of purulent nasal discharge with obstruction B. b. Presence of watery nasal discharge with headache C. c. Presence of watery nasal discharge with obstruction D. d. Presence of purulent discharge with headache MPL: 1.00 Which of the following best describe vasomotor rhinitis A. A lady dentist with nasal obstruction throughout her pregnancy B. A jeepney driver with nasal obstruction and sneezing C. A medical intern with one sided nasal obstruction and headache D. A farmer with nasal obstruction and crusts in the nasal cavity MPL: 0.75

D

29

Rhinitis medicamentosa is due to overuse of A. Oral steroids B. Oral decongestants C. Local steroids D. Local decongestants MPL: 0.75

203

C

30

Which of the following best describe a patient with juvenile angiofibroma A. A 24 y/o female bank teller with epistaxis during office hours B. A 66 y/o male, smoker, with intranasal mass C. A 14 y/o boy, with profuse epistaxis D. A 53 y/o hypertensive female with epistaxis MPL: 0.75

245

C

31

Which is the most critical aspect of management in odontogenic infection: A. Appropriate cidal antibiotic B. Perform aggressive incision and drainage C. Remove the offending tooth D. Work-up to identify possible mediastinal extension MPL: 0.75

297

A

33

A 50-year old female presents clinically with globus hystericus. This is usually associated with a/an: A. A normal PE and barium swallow B. Lesion of the upper esophagus C. Abnormal direct laryngoscopy D. Structure of the upper esophagus MPL: 0.75

392

B

34

A 20-year old sailor has a persistent sorethroat and is found to have a pharyngitis with enlarged tonsils and cervical adenophaty . Gram stain of pharyngeal secretions

344

100

225

shows gram- negative diplococci. These findings support a diagnosis of: A. Streptococcal pharyngitis B. Gonococcal pharyngitis C. Haemophilus influenza D. Bacteroides pharyngitis MPL: 1.00 D

35

C 36 350-351

A 25y/o male consulted at the OPD due to dysphagia of 2 days duration. PPE: (+) trismus, increased salivation, peritonsillar swelling (left) pushing uvula across midline. What is your working diagnosis? A. Acute tonsillitis B. Acute pharyngitis C. Acute tonsillophayngitis D. Peritonsillar abscess MPL: 0.75

346

This lymphoid tissue may be completely removed because of the presence of a capsule: A. Guerlach‟s tonsils B. Pharyngeal tonsil C. Palatine tonsils D. Lingual tonsil MPL: 0.75 A 50 year-old male had barium swallow with the report of aperistalsis, esophageal dilatation, and failure of the lower esophageal spinchter to relax. Which of the following is your primary consideration? A. Presbyesophagus B. Diffuse Esophageal Spasm C. Achalasia D. Scleroderma MPL: 0.50

C

37

471

C

38

Tonsillectomy should be considered for all of the following except: A. Obstructive sleep apnea B. Recurrent peritonsillar abscess C. Asymptomatic tonsillar hyperplasia D. Unilateral tonsillar hyperplasia MPL: 0.75

A

39

Complication of a retropharyngeal abscess may involve: A. Mediastinitis B. Hypoglossal paralysis C. Horner‟s syndrome D. Thyroiditis MPL: 0.25

362

A

40

The region of the pharynx that extends from the base of the skull to the level of hard palate: A. Epipharynx B. Mesopharynx C. Hypopharynx D. Laryngopharynx MPL: 0.50

274

C

41

Majority of FB coins will be trapped in the: A. Gastroesophageal constriction B. Aortic and bronchial constriction in the esophagus C. Cricopharyngeal constriction D. Tonsils MPL: 0.75

352

475

A 1 1/2 month-old baby girl was brought to your clinic because of stridor. There was no associated cough. The patient was delivered to a G8P7 40 year old laundrywoman from payatas via NSD. Birthweight was 6.5lbs with APGAR score of 9 and 10. She was sent home after 2 days with no perinatal complications. She has a good suck with normal swallowing, good cry and weight gain. Stridor is slightly relieved by putting her in a supine position. T=37.2 C ; RR= 40/min. D

43

Your initial impression is: 393-394 A. Congenital subglottic stenosis B. Congenital laryngeal web C. Laryngocoele D. Laryngomalacia MPL: 0.75

D

44

The best thing to do in this patient for your initial evaluation is/are: A. Request for a plain chest x-ray B. Tracheostomy C. Request for CT scan of the neck D. Direct laryngoscopy MPL: 0.75

393

A 6 year-old boy was brought in to the clinic because of cough associated with moderate-grade fever and sorethroat characterized by painful swallowing few days PTC. The patient was sent home with antibiotics, cough syrup and antipyretics. The following day, patient was brought to the ER because of stridor associated with cyanosis. Patient was relieved by sitting up with mouth open and chin forward. He was restless and cyanotic with flaring of alae nasi, substernal and intercostal retractions. T=40C ; RR=36/min. B

45

Your initial impression is: A. Drug allergy B. Acute epiglottitis C. Acute subglottic laryngitis D. Foreign body in laryngotracheobronchial tree MPL: 1.00

399

D

46

The best thing to do in this patient as your initial evaluation and management is/are: A. Chest X-ray B. Tracheostomy C. Laryngoscopy D. Antibiotics MPL: 0.75

399

A 62 year old farmer from Iriga City came in because of dyspnea. His condition started 10 months PTC as hoarseness associated with hemoptysis. One month PTC, he developed persistent dyspnea accompanied by a lump on the right side of the neck. Patient is a chronic alcoholic and smoker. On PE, patient is obese, afebrile, and in respiratory distress. D

47

Your initial impression is: A. Chronic non-specific laryngitis B. Pedunculated vocal cord polyp C. Laryngeal papilloma D. Laryngeal CA MPL: 0.75

C

48

The patient requires: A. Chest X-ray B. CT scan of the neck C. Direct laryngoscopy and biopsy D. Neck node excision biopsy MPL: 0.75

461

461

A 3 year-old girl was taken to the ER because of stridor and cyanosis. Few hours PTC, patient was playing in the sala while her daddy was eating peanut while watching world meeting of families. The patient was restless with audible slap and palpable thud. B

49

Your initial impression is: A. Bronchial foreign body B. Tracheal foreign body C. Tracheomalacia D. Esophageal foreign body MPL: 0.50

484

A

50

Your initial diagnostic procedure is: A. Chest X-ray B. CT scan of the chest C. CT scan of the neck D. Direct laryngoscopy MPL: 0.25

484

A

51

The only bony structure in the laryngeal skeleton is the A. hyoid B. thyoid d. C. cricoid D. epiglottis MPL: 1.00

C

53

The best way to examine the larynx in the OPD is by A. direct laryngoscopy B. palpation C. indirect laryngoscopy D. lateral neck X-rays MPL: 1.00

392

D

54

The most common congenital laryngeal anomaly is A. subglottic stenosis B. vocal fold paralysis C. laryngeal hemagioma D. laryngomalacia MPL: 1.00

384

384-385

A

55

The primary structure preventing laryngeal aspiration is: A. True vocal cords B. False vocal cords C. Epiglottis D. Valecula MPL: 0.75

A

62

Symptoms of mandibular fracture include A. malocclusion B. epistaxis C. a flat face D. lateral neck numbness MPL: 1.00

532

C

63

X-ray evidence of unilateral haziness of the maxillary sinus in a trauma case indicates A. foreign body impaction in the sinus B. total collapse of the sinus walls C. blood in the sinus D. sinusitis MPL: 0.75

528

B

64

The treatment of choice for depressed zygomatic arch fracture is A. Caldwell Luc Operation B. Gillie's Operation C. Forced Duction Operation D. Titanium plate and screw fixation. MPL: 0.25

B

65

Procedure of choice for neglected nasal bone fracture A. ORIF B. Septorhinoplasty C. Closed reduction D. Septoplasty MPL: 0.75

B

66

Tear drop” sign indicative of blow-out fracture is seen best on: A. AP view B. Water‟s view C. Vertex view D. Transorbital view MPL: 0.50

C

67

A simple depressed fracture of the nose is best treated by: A. Simple septorhinoplasty B. Open reduction C. Closed reduction D. Titanium plating

531

B

68

The second most common fracture in the face is: A. nasal bone fracture B. Mandibular fracture C. c. Alveolar ridge fracture D. d. Zygomatic fracture MPL: 1.00

532

B

69

The Le Fort II fracture is also known as A. Guerin fracture B. Pyramidal fracture C. Craniofacial dysjunction D. Rhomboid fracture MPL: 1.00

537

A

70

The Le Fort I fracture is also known as A. Guerin fracture B. Pyramidal fracture C. Craniofacial dysjunction D. Rhomboid fracture MPL: 1.00

537

C

72

Repair of a cleft palate is best done A. soon after birth B. before school age C. before speech develops D. before dating age (adolescent). MPL: 1.00

C

73

The A. B. C.

operation for prominent ear lobe deformity is called: tympanoplasty myringotomy otoplasty

390

535

531

535

288

96

D. auditory meatoplasty MPL: 1.00 C

76

Scar tissue formation may be delayed by using A. antibiotics B. lidocaine injection C. steroids D. epinephrine MPL: 0.75

517

B

77

A free flap derives its blood supply from: A. A neighboring artery in the recipient site B. The capillaries in the recipient site C. The arterial supply in the donor site D. The capillaries in the donor site MPL: 0.75

514

D

78

In wound healing, during the stage of injury, which of the following occurs: A. PMN leukocytes predominate the population of inflammatory cells in wound B. wound is filled with extracellular matrix C. fibroblasts fill up the wound D. formation of hemostatic plug composed of fibrin and platelet MPL: 0.75

B

79

The most usual donor site for split thickness skin graft A. neck B. thigh C. abdomen D. post-auricular area MPL: 0.50

512

B

80

Which of the following grafts has a better chance of “take”(survival of the graft) ? A. Full thickness skin graft B. Split thickness skin graft C. Free bone graft D. Autogenous graft MPL: 0.75

516

B

81

A 54-year-old patient presented with a midline mass that moves with deglutition. Your primary diagnostic impression would be, that maybe it‟s a/an: A. Thyroglossal duct cyst that manifested late in life. B. Thyroid nodule, probably benign C. Thyroid malignancy D. Inflammatory nodule MPL: 0.75

A

82

The most logical question to ask when probing whether the mass is congenital, would be: A. What is the age of the patient? B. Is the mass growing rapidly? C. Is the mass cystic or solid? D. Is there any source of infection? MPL: 1.00

431

C

83

A 30 year old female comes in with fever, pains in the right pre-auricular area. On examination, you see a swollen, tender, erythematous right parotid area. The orifice of the stensen‟s duct is also swollen with minimal purulent material coming out. What is the diagnosis? A. Sialolithiasis B. Chronic sialadenitis C. Acute bacterial sialadenitis D. Mumps MPL: 1.00

319

D

84

A 40 year old male presents with a 3 X 3 cm right pre-auricular mass. FNAB showed pleomorphic adenoma. Which of the following statements is incorrect? A. It is the most common benign neoplasm in the parotid gland B. The tumor has a slow growth pattern C. Treatment of choice is superficial parotidectomy with facial nerve preservation D. Recurrence is not a problem in such a case. MPL: 0.50

327

D

85

A 20 y/o male came to the clinic with a mass at the left submandibular area. Primary consideration as to the nature of the mass would probably be: A. Neoplastic B. Congenital C. Cystic D. Inflammatory MPL: 1.00

319

505

439-440

A 60 y/o female teacher from Iriga City came in for a right pre-auricular mass of 2 months duration. Initially the patient palpated a tender 1 X 0.5 cm mass which progressively enlarged to its present size. On P.E there is 3 X 4 X 2 cm tender, movable, firm to stony hard, nodular mass on the right pre-auricular area. The overlying skin is not indurated and moves freely over the mass. No cervical lymphadenopathy was noted. D

86

The finding in the above patient which supports the possibility of a parotid gland malignancy: A. The age of the patient B. The absence of cervical lymphadenopathy C. The presence of pain and duration of the mass D. The age of the patient and duration of the mass MPL: 0.75 Surgical treatment of choice for thyroglossal duct cyst is A. Marsupialization procedure B. Excision of cyst tract and portion of hyoid bone C. Excision of cyst and tract D. Removal of hyoid bone MPL: 0.50

431

B

91

B

93

The treatment of choice in malignant lymphoma is/are: A. Surgery B. Radiotherapy and/or chemotherapy C. Chemotherapy D. Surgery and/or chemotherapy MPL: 0.50

437

C

94

The earliest sign of carcinoma of the larynx is A. Sore throat B. Dysphagia C. Hoarseness D. Dyspnea MPL: 0.75

444

C

95

The most common histologic type of carcinoma in the head and neck area is A. Adenocarcinoma B. Basal cell carcinoma C. Squamous cell carcinoma D. Sarcoma MPL: 0.75

445

B

96

The most commonly benign salivary gland tumor in children: A. Papilloma B. Hemangioma C. Pleomorphic Adenoma D. Mucoepidermoid carcinoma MPL: 0.50

325

B

97

Knowledge of the lymphatic drainage is important because it will: A. Go to the primary lesion B. Give an idea where the primary lesion is C. Be evident in the histopath D. Distribute lymph as a defense mechanism MPL: 1.00

C

98

A neoplasm that occurs primarily in the parotid gland, more commonly seen in males in the older age group and is also known as papillary cystadenoma lymphomatosum A. Oncocytoma B. Pleomorphic adenoma C. Warthin‟s tumor D. Hemangioma MPL: 0.75

C

99

Sudden facial paralysis associated with vesicles in the external audiotory canal and pinna: A. Melkersson‟s syndrome B. Bell‟s palsy C. Ramsay hunt syndrome D. Schwannoma MPL: 0.75

C

100

1.

The opening of the parotid duct is at level: A. 1st upper molar B. 1st lower molar C. 2nd upper molar D. 2nd lower molar MPL: 1.00

In which of the following conditions is the cornea most opacified? MPL = 0.25 a. Macula b. Leukoma

434

448

327

83

317

c. d.

Nebula Corneal abrasion

2.

A condition that gives painless blurring of vision is: MPL = 1.0 a. Central retinal vein occlusion b. Orbital cellulitis c. Optic neuritis d. scleritis

3.

The most important factor for developing diabetic retinopathy is: MPL = 0.25 a. Duration of the diabetes b. Poor metabolic control of diabetes c. Concomitant renal disease d. Concomitant systemic hypertension

4.

In hyperopia, the axial length is: MPL = 1.0 a. Too short b. Too long c. Just right d. Of no significance

5.

The leading cause of avoidable and reversible blindness worldwide is/are … MPL = 0.5 a. …Cataracts. b. …Corneal leukomas c. …Trauma. d. …Glaucoma

6.

The most useful chart attractive to preschool illiterate children: MPL = 0.5 a. Snellen Chart b. Illiterate „E‟ Chart c. Allen Chart d. ETDRS Chart

7.

One advantage of Direct Ophthalmoscopy over Indirect Ophthalmoscopy is: MPL = 1.0 a. Stereoscopy b. Magnification c. Light Intensity d. Wider Field of Vision

8.

Flouresecin dye when viewed using a cobalt blue light will appear as: MPL = 0.5 a. Luminous orange b. Luminous red c. Luminous blue d. Luminous green

9.

Blind spot on a normal visual field is also known as: MPL = 1.0 a. Scotoma b. Epiphora c. Diplopia d. Asthenopia

10. Most accidental eye injuries can be prevented by… MPL = 0.5 a. …a good pair of protective goggles b. … a good surgical mask c. ... common sense d. all of the above 11. The most important consideration in Testing Near Acuity: MPL = 0.25 a. Age b. Testing Distance c. Literacy d. Stereoscopic Vision 12. Determination of Visual Field Extent is best demonstrated by: MPL = 0.25 a. Octopus Perimetry b. Automated Keratometry c. Amsler Grid Testing d. Confrontation Fields Test 13. In performing Direct Ophthalmoscopy, the first element that must be observed: MPL = 0.5 a. Media Clarity b. Red reflex c. Macular changes d. Cup / Disc features 14. In the three-part step-wise sequence in external eye examination, one procedure that is not usually included: MPL = 1.0 a. Inspection b. Auscultation c. Palpation d. Percussion

15. The patient was not able to see hand movement. What is the next step to check his visual acuity?MPL= 0.5 a. Check light perception* b. Have the patient walk closer to the Snellen chart and ask if he can see the biggest letter c. Do counting fingers d. Check color perception test 16. A patient is only able to read the first line of the Jaeger chart. His near visual acuity is recorded as…MPL=0.3 a. J16 b. J16 -2 lines* c. J16 +2 lines d. J16 +2 17. In assessing facial sensation, the three branches of the cranial nerve V that is being tested are…MPL=0.3 a. Supraorbital n., infraorbital n., intraorbital n. b. Facial n., abducens n., trochlear n. c. Frontal n., zygomatic n., lacrimal n. d. Ophthalmic n., maxillary n., mandibular n.* 18. Which of the following is/are true of the corneal blink reflex? MPL=0.3 a. Test is done by touching the eyelashes lightly with a wisp of cotton b. The afferent arm of the reflex is the trigeminal nerve* c. The efferent arm of the reflex is the optic nerve d. Visual startle reflex should be stimulated 19. Which of the following is a term used to describe normal binocular eye movement in the same direction? MPL=0.5 a. Ductions b. Versions* c. Vergence d. Convergence 20. Giant papillary reaction is common in… MPL = 1.0 a. Contact lens overuse b. Preseptal cellulitis c. Orbital cellulitis d. Chalazion 21. Ophthalmia Neonatorum is usually a bacterial cause of conjunctivitis: MPL =0.3 a. True b. False c. Only rarely of bacterial cause d. Real causative organism is unknown 22. The spread of epidemic viral conjunctivitis is best curtailed by: MPL =0.25 a. Putting prophylactic medication on the eyes b. Avoiding crowded places c. Avoid touching one‟s own eyes d. Frequent hand washing 23. Which is not an anatomic locale description of the conjunctiva? MPL =0.5 a. Tarsal b. Fornix c. Limbal d. Bulbar 24. Adenoviral conjunctivitis may cause: MPL =0.3 a. a true membrane on the tarsal conjunctiva b. a pseudomembrane c. never forms any membrane d. may develop keratinized conjunctiva 25. Which of the following statements regarding the use of steroid eye drops for red eyes is correct? MPL =0.3 a. Patients may self medicate with steroids when they have red eyes. b. Steroids may be used so long as the dosage is less than 4 times a day. c. The possible complications of steroid use include Glaucoma AND Cataract formation d. Ocular complications of steroid eye drops are independent of dosage and duration of use 26. The medication specifically used against adenovirus conjunctivitis currently is: MPL =0.3 a. Steroid antibiotic combination eye drops. b. 4th generation flouroquinolone eye drops (moxifloxacin) c. Interferon eye drops d. There is no specific medication against adenovirus 27. The definitive management of gonococcal conjunctivitis is: MPL =0.3 a. Tobramycin eye drops every hour to affected eye for 10 days. b. Ceftriaxone 1gram intramuscular injection once only. c. Gentamycin intravenous, 200mg q 6hrs for one week. d. Oral amoxicillin, 500mg t.i.d. for one week 28. Which of the following conjunctivitides is almost always bilateral? MPL =0.5 a. Viral b. Bacterial

c. d.

Toxic Allergic

29. Fusion a. b. c. d.

of the bulbar and tarsal conjunctiva as a result of prolonged inflammation and scarring is termed: MPL =0.3 Ankyloblepharon. Symblepharon Eryblepharon Blepharitis

30. Painful a. b. c. d.

blurring of vision can be due to: MPL = 1.0 Hypertensive Retinopathy Acute angle closure glaucoma Vitreous hemorrhage Mature Cataract

31. Glaucoma is characterized by: MPL = 0.25 a. Blurred disc margin b. Exudates c. Conjunctival discharge d. Damage to optic nerve head 32. Secondary glaucoma may occur in: MPL = 0.25 a. Hyphema b. Acute Bacterial conjunctivitis c. Central retinal vein occlusion d. Retinal detachment 33. The hallmark of Grade IV hypertensive retinopathy associated with malignant hypertension is: MPL = 0.25 a. silver-wiring of arterioles b. macular „star‟ (hard exudates) c. optic disc edema d. cotton-wool spots 34. The systemic condition that presents with the most complications in the posterior pole of the eye: MPL = 0.25 a. Tuberculosis b. Diabetes Mellitus c. Allergy d. Blood dyscrasias 35. In testing for strabismus, the corneal light reflex did not fall at the same relative position on each eye instead it was located at the temporal borders of the pupils, the patient has: MPL = 0.5 a. Orthophoria b. Esotropia c. Exotropia d. Hypertropia 36. In cover testing, the uncovered eye moves inward to fixate, the patient has: MPL = 0.5 a. Orthophoria b. Esotropia c. Exotropia d. Hypertropia 37. A 5 year old child was brought to an ophthalmologist because his family had noticed a strange, white reflection that appeared in one of his eyes in photographs while the rest have red eyes due to the flash. Further examination disclosed Retinoblastoma. This white pupil is known as: MPL=0.5 a. pseudohypopyon b. leucocoria c. Hyphema d. Red eye e. Strabismus 38. Computed Tomogram of the above child showed a unilateral tumor filling the left eye. Nasal third of the tumor is calcified. Impression of Retinoblastoma is confirmed with the following histologic features: MPL=0.5 a. Islands of blue cells in a sea of pink necrosis b. Cuboidal cells circled around a central luman: Flexner-Wintersteiner rosettes c. Flower like grouping of tumor cells which look like photoreceptors: Fleurettes d. All of the above e. B and c only 39. A 23/M consulted the OPD because anterior eyelid crusting of 3 days duration. The resident requested for a gramstain which revealed Gram-positive cocci in clusters. Which statement is consistent with the impression of Staphylococcal blepharitis? MPL = 0.5 a. Coagulase production specifies etiologic agent to be Staphylococcus aureus b. Catalase negative reaction identifies normal flora Staphylococcus epidermidis c. All staphylococcus species are catalase negative d. B-lactamase production increases susceptibility to penicillins 40. Which of the following viruses is transmissible even after medical instrumentation is cleaned with alcohol? MPL = 0.5 a. Herpes simplex virus b. Adenovirus

c. d.

Human immunodeficiency virus Epstein-Barr virus

41. A newborn was admitted to the NICU because of mucopurulent eye discharge at birth. The mother had a documented chlamydial genito-urinary tract infection. What bacterial features are consistent with the neonatal ocular finding? MPL = 0.5 a. Elementary bodies stain blue with Giemsa in contrast to the purple of the host cell cytoplasm b. Chlamydia trachomatis strains synthesize folates thus are resistant to sulfonamides c. The outer cell wall resembles the cell-wall of gram-negative bacteria d. Chlamydiae are obligate intracellular parasites that need to exist within host cells to synthesize ATP 42. A 50/M farmer consulted the OPD because of right corneal opacity of 3 weeks duration. He claimed his right eye was hit by a palay strand while harvesting. Clinical history alone leads to a strong suspicion of fungal keratitis. Which of the following statements is a recommended diagnostic management of fungal keratitis? MPL = 0.5 a. Fungi can easily be visualized using the routine hematoxylin and eosin (H&E) preparation b. The cell wall polysaccharides are uniquely stained by periodic acid-Schiff (PAS) c. The gold standard for morphologic classification is blood agar at room temperature d. Corneal scrapings are best collected at the center of the main lesion rather than the small satellite lesions 43. A 43/F consulted because of a sessile mass at the limbus. The excision biopsy revealed squamous papilloma. Which of the following is true about the association of the Human Papilloma Virus (HPV) with squamous papilloma? MPL = 0.5 a. HPV DNA induces cellular necrosis leading to a secondary papilloma growth b. HPV subtypes 6 and 11 have been identified in hypertrophic papillomas c. HPV DNA induces cellular proliferation and can lead to malignancy d. HPV RNA integrates with epithelial genome leading to squamous cell carcinoma 44. A 30/M consulted the OPD because of foreign body sensation with mucopurulent discharge of 6 days duration. A detailed slit-lamp biomicroscopic examination demonstrated clinical features of trachoma. Which are features of an adult trachoma compared to a neonatal chlamydial conjunctivitis is TRUE? MPL = 0.5 a. The amount of mucopurulent discharge is greater b. Membranes develop almost immediately c. The percentage of intracytoplasmic inclusions is greater d. Follicular response is greater 45. Condition in which there is discoloration of the eyelashes MPL = 0.5 a. Madarosis b. Proptosis c. Trichiasis d. Poliosis 46. The condition in which an extra row of lashes exists is MPL =0.5 a. Trichiasis b. Distichiasis c. Entropion d. Ectropion 47. A white pupillary reflex is called: MPL = 1.0 a. leukoma b. glaucoma c. leukocoria d. anisocoria 48. An infant born less than 31 weeks, weighing less than 1500g (3 lb 5 oz) plus exposure to supplemental oxygen, one should screen the eyes for: MPL = 1.0 a. Persistent Hyperplastic Primary Vitreous (PHPV) b. congenital cataract c. a retinal astrocytoma d. Retinopathy of Prematurity (ROP) 49. This is the second most common mode of presentation in Retinoblastoma which account for about 20 % of cases: MPL =0.5 a. leukocoria b. secondary glaucoma c. proptosis d. strabismus 50. The following are treatment options for Retinoblastoma except MPL = 1.0 a. Cryotherapy b. Radio Therapy c. Enucleation d. Evisceration 51. The following are correct in visual acuity testing in a child EXCEPT MPL=0.5 a. At 6 months, test for ocular fixation and ocular movement b. At 4 years old, test with tumbling “E” chart picture chart c. At 8 years old, test with an Amsler grid chart d. At 16 years old, test with a Snellen chart 52. Leukocoria caused by the presence of red cells in the vitreous secondary to birth trauma MPL=0.5

a. b. c. d.

Retinopathy of prematurity Persistent Hyperplastic Primary Vitreous Vitreous Hemorrhage Posterior Uveitis

53. A biconves, avascular, colorless structure that focuses light rays to the retina, an opacity of which will cause leukocoria MPL=0.5 a. Lens b. Vitreous c. Retina d. Optic Nerve 54. Ophthalmological examination that would identify presence of an intraocular mass MPL=0.5 a. Visual acuity testing b. Indirect Ophthalmoscopy c. Pupillary examination d. Sit lamp examination 55. The most common presenting sign of retinoblastoma is MPL=1.0 a. Red eye b. Visual loss c. White pupil d. Strabismus 56. Due to the inheritance pattern of retinoblastoma, a vital part of treatment would incude: MPL=0.5 a. Psychological counseling b. Genetic counseling c. Psychotherapy d. Rehabilitation counseling 57. A surgical procedure involving removal of intraocular contents: MPL=0.5 a. enucleation b. evisceration c. exenteration a. retrobulbar alcohol 58. Management of CRAO includes MPL =0.5 a. “Brown bagging” b. Alternate compression and decompression of the globe c. Oxygen inhalation d. A and B only e. B and C only 59. Characteristic findings in Central Retinal Artery Occlusion MPL = 0.25 a. Cotton wool spots b. Cherry red spot in the macula c. Vitreous floaters d. Papilledema 60. The TRUE statement regarding chemical burns MPL = 1.0 a. Acid burns cause more extensive damage than alkali burns b. The first thing to do in cases of chemical burns is to check the visual acuity c. Neutralization should be achieved, so that an alkali burn can be treated by instilling an acidic substance and vice versa d. In copious irrigation of the eye, the fluid used need not be sterile, provided the chemical is diluted properly 61. EOM movement of one eye is called MPL =0.5 a. Ductions b. Versions c. Nystagmus d. Diplopia 62. Absence of venous pulsations on funduscopy MPL =0.3 a. May indicate increase intracranial pressure b. May be normal c. Is a sign of glaucoma d. A and b 63. To examine the optic nerve binocularly, use MPL = 0.3 a. Indirect ophthalmoscope b. Non contact fundus lens c. Intraocular lens d. A and B 64. What ancillary procedures should you do if suspecting optic nerve toxicity? MPL = 0.25 a. Visual field exam b. Visual evoked potential c. Color test d. All of the above

65. One of a. b. c. d.

the most common reasons for Neuroophtha referral … MPL = 1.0 …unexplained vision loss …cataracts …retinal disorders none of the above

66. Which of the following is true regarding viewing the fundus… MPL = 1.0 a. Indirect ophthalmoscopes provides a monocular view of the fundus b. Direct ophthalmoscopes provides a Binocular view of the fundus c. Direct ophthalmoscopy provides greater detail of the fundus d. none of the above 67. What is the most important adverse effect of local anesthetic overdosage? MPL = 0.25 a. bronchoconstriction b. renal failure c. convulsion d. skin rashes 68. Cortocosteroid stops the inflammatory process by inhibiting prostaglandin synthesis through the following mechanism: MPL = 1.0 a. blockage of the enzyme cyclooxygenase b. blockage of the enzyme phospholipase A c. blockage of GABA pain receptors d. regulation of endorphin levels in the blood 69. Which of the following statements regarding papilledema is not true? MPL =0.3 a. Loss of venous pulsation is always present b. Symptoms accompanying papilledema may include visual loss and diplopia c. Nerve fiber layer hemorrhages around the disc are often present d. Blurring of the peripapillary vessels 70. Most common visual field finding in papilledema MPL = 0.3 a. Central scotoma b. Cecocentral scotoma c. Altitudinal defect d. Enlargement of the blind spot 71. Typically the onset of visual loss in demyelinating optic neuritis is noted MPL = 0.25 a. Upon arising in the morning b. Mid-day c. At the end of the day d. There is no temporal pattern for visual loss 72. One of a. b. c. d.

the following findings is not a characteristic of true disc edema MPL = 0.25 capillary dilation abnormal branching of vessels around the disc disc hemorrhages blurring of disc vessels along the margin

73. Most common cause of optic disc swelling in children MPL = 0.3 a. Infectious optic neuritis b. Demyelinating optic neuritis c. Leber optic neuropathy d. Non arteritic anterior ischemic optic neuropathy 74. One of a. b. c. d.

the following is not a characteristic finding of optic neuritis in children MPL = 0.25 Often present simultaneously as a bilateral condition More often anterior in location with optic disc edema seen on exam Occurs 1-2weeks after a known or presumed viral infection Always resistant to steroids

75. Most common condition associated with nonarteritic anterior ischemic optic neuropathy MPL = 0.5 a. Diabetes mellitus b. Carotid artery disease c. Coagulopathies d. Hypertension 76. Which of the following is the most common optic neuropathy in patients over the age of 50? MPL = 0.3 a. Compressive b. Demyelinating c. Traumatic d. Nonarteritic anterior ischemic optic neuropathy 77. 10. An obese 26 year old woman presents with headaches and transient visual obscurations. Bilateral papilledema is documented. The next step should be which of the following MPL = 0.3 a. advised weight reduction b. do lumbar tap c. Start on acetazolamide d. Do CT or MRI of the brain

78. Exophthalmos is a term used specifically in which of the following conditions? MPL = 1.0 a. Thyroid disease b. Anemia c. Keratoconus d. Myopia 1. The most commonly observed Cumulative Trauma disorder is

A. B. C. D.

De Quervains Trigger finger Carpal tunnel syndrome Tennis elbow

2. The ff. is an indirect cost of injury among workers

A. B. C. D.

medical Cost Lost Time Wages Training replacements Compensation Premiums

3.The ff. is an indication for surgical release of CTS

A. B. C. D.

Severe causalgia Weak Abductor Pollicis brevis Atrophy of hypothenar eminence edema on the wrist

4. The ff. is a feature of Impingement Syndrome

A. B. C. D.

Limited Passive ROM of the shoulder Pain active on shoulder flexion Limited active abduction of the shoulder Positive Spurling sign

5. In Reflex sympathetic dystrophy the ff. is true

A. B. C. D.

Pathology originates at the fingers Autonomic changes Complete shoulder ROM Splinting the shoulder helps a lot

6. The most common lump on the hand is

A. B. C. D.

Trigger finger Ganglion cyst Tendenitis Dupuytrens contracture

7. The Following can lead to Frozen Shoulder EXCEPT

A. B. C. D.

Rotator cuff Tear Bicipital tendenitis Carpal Tunnel Syndrome Trigger finger

8. Avascular necrosis of the Lunate

A. B. C. D.

Panners Keinbocks Dupuytrens Osgood

9. A worker comes with a lump on the dorsal elbow, it is movable, soft with tenderness. He has a habit on putting his weight at the elbow , he has

A. B. C. D.

Golfer‟s elbow Tennis elbow Tendinitis Student‟s elbow

10. The best thing to do in acute tendenitis is

A. B. C. D.

To stretch it to increase tensile strength To rest it in a splint To move it to prevent contracture To apply hot moist pack

11. The grip strength of a dominant hand is usually stronger than the non dominant hand by

A. B.

10 lb.

C. 30 lbs

20 lb.

D. 40 lbs

12. A functional non organic sensation deficit is evident if there is

A. B. C. D.

vibratory deficit in the upper arm sensory deficit in the whole leg numbness of the thumb numbness on the right side of the face

13. To prevent overvaluation and subjection to potential risk of tests. The following will suggest a non organic findings in back pain

A. B. C. D.

Pain on straight leg raising at 30 degrees light pinch tenderness of skin on vast area of the back Sensory deficit on the lateral thigh weak extensor hallucis longus

14. The most common type of muscular dystrophy is

A. B. C. D.

Limb Girdle Duschenne Facioscapulo-humeral Beckers

15. Cramps is the usual complain of

A. B. C. D.

Duschenne Myotonic SMA Limb Girdle

16. Spinal dysraphism can be detected in utero after 18 weeks by taking the

A. B. C. D.

Amniotic C -reactive protein Amniotic Alpha feto protein Amniotic phosphofructokinase Amniotic amylase

17. A positive Gowers sign is

A. B. C. D.

Pathognomonic in muscular dystrophy Present in pelvic and proximal leg weakness seen in hip flexion contracture a feature of McArdle‟s disease

18. Ischemic Compression Test is for

A. B. C. D.

Duschenne MD Myotonia SMA McArdle‟s Disease

19. A dive bomber sound on EMG is seen in

A. B. C. D.

Beckers MD Myotonia Spinal Dysraphism SMA

20. Fasciculation‟s of the tongue and limb muscles is frequently seen in

A. B.

Beckers MD Myotonia

C. Spinal Dysraphism D. SMA

21. Most of children with this condition cannot reach adulthood except

A. B. C. D.

Duschenne Fascioscapulohumeral dystrophy Werdnig Hoffman (Acute) Pompe‟s Disease

22. The most common cause of death of children with myopathies is

A. B.

Renal Problem

C. Pulmonary problem

Weakness

D. Myocardial infarction

23. This reflex is integrated in the spinal cord

A. B. C.

D.

Asymmetric tonic neck reflex Optical righting Flexor withdrawal Landau reflex

24. This reflex develops mouth opening, helps find the breast and develops various tongue position A. Sucking reflex B. Rooting reflex C. Automatic head turning D. Demand feeding reflex 25. Asymmetric Tonic Neck reflex A. Enables each side of the body to be used separately B. Coordinates with tonic labyrinthine reflex for turning the body C. Integrated in the cortex D. Appears at six month of age 26. Patient with Spina bifida should be examined for A. Presence of strabismus B. Presence of Mongolian spot C. Presence of hip dislocation D. Presence of hormonal problem 27. . This instrument is used to measure range of motion of joints: A. tape measure C. tachometer B. goniometer D. planometer 28. . Which of the following is considered as a deep heating modality: A. heating pads C. ultrasound B. whirlpool baths D. paraffin baths 29. On stimulation the baby extend and abduct the limbs followed by flexion and adduction A. If this is a normal baby he is less than four months B. This is a reflex integrated in the midbrain C. This reflex will not interfere in balance and rolling over D. This is a normal reaction in baby of all ages 30. . Stroke A. B. C. D.

rehabilitation may be started: as soon as the patient‟s neurological and medical condition stabilizes after 2 weeks of onset neurologic symptoms as soon as the diagnosis of a stroke is made as soon as a motor grade of at least 2/5 in the affected extremities are noted

31.This reflex is integrated in the spinal cord A. Asymmetric tonic neck reflex B. Optical righting C. Flexor withdrawal D. Landau reflex 32. This reflex develops mouth opening, helps find the breast and develops various tongue position A. Sucking reflex B. Rooting reflex C. Automatic head turning D. Demand feeding reflex 33.Asymmetric Tonic Neck reflex A. Enables each side of the body to be used separately B. Coordinates with tonic labyrinthine reflex for turning the body C. Integrated in the cortex D. Appears at six month of age 34. At four months of age the child is expected to A. Sit with good balance B. Grasp objects dangled in front of face C. Increased flexion of the trunk D. Rolls over from supine 35. A child of 3 years is expected to A. Roller skates B. Dresses without supervision C. Walks up stairs alternating feet D. Skips 36. A dyskenitic Cerebral palsy is frequently associated with A. Bilirubin encephalopathy

B. C. D.

Anoxic spells Birth Trauma Hypotonic type

37.True of a geriatric individual  Most of them are institutionalize  They are not interested in sexual activities  They prefer to be independent  They wanted to live with adult children 38.Some of the changes in the elderly is A. Decline in the number of motor units B. Increase in muscle mass C. Preservation of vibratory perception D. Increase in visual acuity 39.Crystallized intelligence is preserved in elderly this is A. Judgment B. Abstract thinking C. comprehension D. Reasoning 40. Elderly has tendency to fall. One reason is a sudden drop of blood pressure which can be due to A. Failure to take antihypertensive B. Sudden change in position or abrupt standing C. Hearing loss D. Visual loss 41.Exercise prescription for an osteoporotic patient is A. Swimming B. Jogging C. Walking D. Bicycling 42.Osteoporosis has an earlier onset in women . And this is usually evident during A. 1 to 3 years after menopause B. 4 to 6 years after menopause C. 5 to 7 years after menopause D. 10 years after menopause 43.Elderly patient is prone to bed sore because of A. Loss of subcutaneous fat B. More prominent blood vessels C. Dementia D. Increased heat dissipation 44. Cardiorespiratory changes in elderly A. Lower maximal heart rate B. Lower total lung capacity C. Less Reserve volume D. Less cardiac output 45.To prevent hypotension in elderly it is best to observe A. Restriction of salt intake B. Keep him in bed most of the time C. Standing slowly while holding to a bar D. Exercise upon standing 46. A spinal cord injury with weaker upper extremities than lower extremities is A. A posterior cord syndrome B. Central cord syndrome C. A bilateral brachial plexus injury D. A hemi section of the spinal cord 47. An SCI patient states that he has intact sensation up to the umbilicus, without movement over his legs. This means A. His sensory level is T10 B. His corresponding motor level is T12 C. His lesion is over T10 vertebra D. He is a quadriplegic 48.The most common site of heterotrophic ossification among burn patient is at the A. Hip B. Knee C. Elbow D. Shoulder

49.A bedridden quadriplegic has painful inflamed thigh . The working diagnosis is DVT , it is best to differentiate this with A. Fracture of the hip B. Heterotopic ossification C. Osteomylitis D. TB arthritis 50.The most common tumor of the bone A. Osteosarcoma B. Multiple Myeloma C. Metastatic D. Osteod osteoma 51.The most common cause of Traumatic brain injury is A. Vehicular accident B. Fall C. Gunshot wound D. Sports injury 52.A C-curve scoliosis is common in A. Idiopathic scoliosis B. Scoliosis secondary to leg length discrepancy C. Scoliosis due to hemivertebra D. Paralytic scoliosis 53.Physical finding in scoliosis on convexity side A. Prominent front chest B. Rotation of the spine C. Lower shoulder level D. Prominent scapula 54. .Physical finding in scoliosis on concavity side A. Rotation of the vertebral body B. Prominent front chest C. Wider rib Space D. Better chest excursion 55.. This syndrome of massive sympathetic discharge associated with the SCI patients with lesions at T6 level, characterized by headache, hypertension, diaphoresis and reflex bradycardia is known as: A. areflexia C. autonomic dysreflexia B. sacral sparing D. cauda equina 56. This is the most important aspect to consider in the management of decubitus ulcer: A. relief of pressure C. use of hot packs B. use of transcutaneous nerve stimulation D. muscle strengthening 57. What is the key muscle for testing C7 myotome, according to the American Spinal Injury Association? A. triceps C. biceps B. deltoids D. first dorsal interossei 58. . This level is generally the highest level of injury at which spontaneous ventilation can be sustained. Injuries above this level generally require mechanical ventilation: A. C4 C. C6 B. C5 D. C7 59. This is referred to as an ectopic bone formation occurring within 6 months after spinal cord injury: A. heterotopic ossification C. osteoporosis B. degenerative joint disease D. compression deformities of the spine 60. This is generally considered the drug of choice for spasticity in SCI: A. Diazepam C. Dantrolene sodium B. Baclofen D. Clonidine 61. The most commonly observed Cumulative Trauma disorder is A. De Quervains B. Trigger finger C. Carpal tunnel syndrome D. Tennis elbow 62. The ff. is an indirect cost of injury among workers

A. B. C. D.

medical Cost Lost Time Wages Training replacements Compensation Premiums

63.The ff. is an indication for surgical release of CTS

A. B. C. D.

Severe causalgia Weak Abductor Pollicis brevis Atrophy of hypothenar eminence edema on the wrist

64. The ff. is a feature of Impingement Syndrome

A. B. C. D.

Limited Passive ROM of the shoulder Pain active on shoulder flexion Limited active abduction of the shoulder Positive Spurling sign

65. In Reflex sympathetic dystrophy the ff. is true

A. B. C. D.

Pathology originates at the fingers Autonomic changes Complete shoulder ROM Splinting the shoulder helps a lot

66. The most common lump on the hand is

A. B. C. D.

Trigger finger Ganglion cyst Tendenitis Dupuytrens contracture

67. The Following can lead to Frozen Shoulder EXCEPT

A. B. C. D.

Rotator cuff Tear Bicipital tendenitis Carpal Tunnel Syndrome Trigger finger

68. Avascular necrosis of the Lunate

A. B. C. D.

Panners Keinbocks Dupuytrens Osgood

69. A worker comes with a lump on the dorsal elbow, it is movable, soft with tenderness. He has a habit on putting his weight at the elbow , he has

A. B. C. D.

Golfer‟s elbow Tennis elbow Tendinitis Student‟s elbow

70. The best thing to do in acute tendenitis is

A. B. C. D.

To stretch it to increase tensile strength To rest it in a splint To move it to prevent contracture To apply hot moist pack

71. The grip strength of a dominant hand is usually stronger than the non dominant hand by

A. B.

10 lb.

C. 30 lbs

20 lb.

D. 40 lbs

72. A functional non organic sensation deficit is evident if there is

A. B. C. D.

vibratory deficit in the upper arm sensory deficit in the whole leg numbness of the thumb numbness on the right side of the face

73. To prevent overvaluation and subjection to potential risk of tests. The following will suggest a non organic findings in back pain A. Pain on straight leg raising at 30 degrees B. light pinch tenderness of skin on vast area of the back C. Sensory deficit on the lateral thigh D. weak extensor hallucis longus 74. The most common type of muscular dystrophy is

A.

Limb Girdle

B. C. D.

Duschenne Facioscapulo-humeral Beckers

75. Cramps is the usual complain of

A. B. C. D.

Duschenne Myotonic SMA Limb Girdle

76. Spinal dysraphism can be detected in utero after 18 weeks by taking the

A. B. C. D.

Amniotic C -reactive protein Amniotic Alpha feto protein Amniotic phosphofructokinase Amniotic amylase

77. A positive Gowers sign is

A. B. C. D.

Pathognomonic in muscular dystrophy Present in pelvic and proximal leg weakness seen in hip flexion contracture a feature of McArdle‟s disease

78. Ischemic Compression Test is for

A. B. C. D.

Duschenne MD Myotonia SMA McArdle‟s Disease

79. A dive bomber sound on EMG is seen in

A. B. C. D.

Beckers MD Myotonia Spinal Dysraphism SMA

80. Fasciculation‟s of the tongue and limb muscles is frequently seen in A. Beckers MD C. Spinal Dysraphism B. Myotonia D. SMA 81. Most of children with this condition cannot reach adulthood except

A. B. C. D.

Duschenne Fascioscapulohumeral dystrophy Werdnig Hoffman (Acute) Pompe‟s Disease

82. The most common cause of death of children with myopathies is

A. B. C. D.

Renal Problem C. Pulmonary problem Weakness D. Myocardial infarction

83. This reflex is integrated in the spinal cord A. Asymmetric tonic neck reflex B. Optical righting C. Flexor withdrawal D. Landau reflex 84. This reflex develops mouth opening, helps find the breast and develops various tongue position A. Sucking reflex B. Rooting reflex C. Automatic head turning D. Demand feeding reflex 85. Asymmetric Tonic Neck reflex A. Enables each side of the body to be used separately B. Coordinates with tonic labyrinthine reflex for turning the body C. Integrated in the cortex D. Appears at six month of age 86. Patient with Spina bifida should be examined for A. Presence of strabismus

B. Presence of Mongolian spot C. Presence of hip dislocation D. Presence of hormonal problem 87. . This instrument is used to measure range of motion of joints: A. tape measure C. tachometer B. goniometer D. planometer 88. . Which of the following is considered as a deep heating modality: A. heating pads C. ultrasound B. whirlpool baths D. paraffin baths 89. On stimulation the baby extend and abduct the limbs followed by flexion and adduction A. If this is a normal baby he is less than four months B. This is a reflex integrated in the midbrain C. This reflex will not interfere in balance and rolling over D. This is a normal reaction in baby of all ages 90. . Stroke A. B. C. D.

rehabilitation may be started: as soon as the patient‟s neurological and medical condition stabilizes after 2 weeks of onset neurologic symptoms as soon as the diagnosis of a stroke is made as soon as a motor grade of at least 2/5 in the affected extremities are noted

91.This reflex is integrated in the spinal cord A. Asymmetric tonic neck reflex B. Optical righting C. Flexor withdrawal D. Landau reflex 92. This reflex develops mouth opening, helps find the breast and develops various tongue position A. Sucking reflex B. Rooting reflex C. Automatic head turning D. Demand feeding reflex 93.Asymmetric Tonic Neck reflex A. Enables each side of the body to be used separately B. Coordinates with tonic labyrinthine reflex for turning the body C. Integrated in the cortex D. Appears at six month of age 94. At four months of age the child is expected to A. Sit with good balance B. Grasp objects dangled in front of face C. Increased flexion of the trunk D. Rolls over from supine 95. A child of 3 years is expected to A. Roller skates B. Dresses without supervision C. Walks up stairs alternating feet D. Skips 96. A dyskenitic Cerebral palsy is frequently associated with A. Bilirubin encephalopathy B. Anoxic spells C. Birth Trauma D. Hypotonic type 97.True of a A. B. C. D.

geriatric individual Most of them are institutionalize They are not interested in sexual activities They prefer to be independent They wanted to live with adult children

98.Some of the changes in the elderly is A. Decline in the number of motor units B. Increase in muscle mass C. Preservation of vibratory perception D. Increase in visual acuity 99.Crystallized intelligence is preserved in elderly this is A. Judgment B. Abstract thinking C. comprehension D. Reasoning 100. Elderly has tendency to fall. One reason is a sudden drop of blood pressure which can be due to A. Failure to take antihypertensive

B. C. D.

Sudden change in position or abrupt standing Hearing loss Visual loss

101.Exercise prescription for an osteoporotic patient is A. Swimming B. Jogging C. Walking D. Bicycling 102.Osteoporosis has an earlier onset in women . And this is usually evident during A. 1 to 3 years after menopause B. 4 to 6 years after menopause C. 5 to 7 years after menopause D. 10 years after menopause 103.Elderly patient is prone to bed sore because of A. Loss of subcutaneous fat B. More prominent blood vessels C. Dementia D. Increased heat dissipation 104. Cardiorespiratory changes in elderly A. Lower maximal heart rate B. Lower total lung capacity C. Less Reserve volume D. Less cardiac output 105.To prevent hypotension in elderly it is best to observe A. Restriction of salt intake B. Keep him in bed most of the time C. Standing slowly while holding to a bar D. Exercise upon standing 106. A spinal cord injury with weaker upper extremities than lower extremities is A. A posterior cord syndrome B. Central cord syndrome C. A bilateral brachial plexus injury D. A hemi section of the spinal cord 107. An SCI patient A. B. C. D.

states that he has intact sensation up to the umbilicus, without movement over his legs. This means His sensory level is T10 His corresponding motor level is T12 His lesion is over T10 vertebra He is a quadriplegic

108.The most common site of heterotrophic ossification among burn patient is at the A. Hip B. Knee C. Elbow D. Shoulder 109.A bedridden A. B. C. D.

quadriplegic has painful inflamed thigh . The working diagnosis is DVT , it is best to differentiate this with Fracture of the hip Heterotopic ossification Osteomylitis TB arthritis

110.The most common tumor of the bone A. Osteosarcoma B. Multiple Myeloma C. Metastatic D. Osteod osteoma 111.The most common cause of Traumatic brain injury is A. Vehicular accident B. Fall C. Gunshot wound D. Sports injury 112.A C-curve scoliosis is common in A. Idiopathic scoliosis B. Scoliosis secondary to leg length discrepancy C. Scoliosis due to hemivertebra D. Paralytic scoliosis 113.Physical finding in scoliosis on convexity side A. Prominent front chest

B. C. D.

Rotation of the spine Lower shoulder level Prominent scapula

114. .Physical finding in scoliosis on concavity side A. Rotation of the vertebral body B. Prominent front chest C. Wider rib Space D. Better chest excursion 115.. This syndrome of massive sympathetic discharge associated with the SCI patients with lesions at T6 level, characterized by headache, hypertension, diaphoresis and reflex bradycardia is known as: A. areflexia C. autonomic dysreflexia B. sacral sparing D. cauda equina 116. This is the most important aspect to consider in the management of decubitus ulcer: A. relief of pressure C. use of hot packs B. use of transcutaneous nerve stimulation D. muscle strengthening 117. What is the key muscle for testing C7 myotome, according to the American Spinal Injury Association? A. triceps C. biceps B. deltoids D. first dorsal interossei 118. . This level is generally the highest level of injury at which spontaneous ventilation can be sustained. Injuries above this level generally require mechanical ventilation: A. C4 C. C6 B. C5 D. C7 119. This is referred to as an ectopic bone formation occurring within 6 months after spinal cord injury: A. heterotopic ossification C. osteoporosis B. degenerative joint disease D. compression deformities of the spine 120. This is generally considered the drug of choice for spasticity in SCI: A. Diazepam C. Dantrolene sodium B. Baclofen D. Clonidine _____ 1. A female cancer patient is in constant pain which is severe, radiating, shooting and electric like in character. Most likely cause of her pain is: A. The enlarged mass itself B. Compression of a nerve by the enlarged mass C. Bone metastasis D. Treatment related _____ 2. Gate‟s Control Theory of pain is the basis for all the following non pharmacologic pain management EXCEPT: A. Transcutaneous electrical nerve stimulation B. Cold compress C. Massage D. Biofeedback _____ 3. A peripheral type of pain: A. Myofascial pain syndrome B. C. Post herpetic neuralgia C. Reflex sympathetic dystrophy D. D. Tension headache _____ 4. Management of pain due to excision of mass in the anterior abdominal wall: A. Non-steroidal anti inflammatory agents (NSAIDs) B. Opioids C. Opioids and NSAIDs D. NSAIDs, opioids plus anti depressants _____ 5. Chronic use or prolonged intermittent administration of this opioids. Results in neurotoxicity due to its metabolite. A. Meperidine C. Fentanyl B. Morphine D. Oxycodone _____ 6. Major toxicity of short term use of NSAIDs: A. Exacerbation of hypertension B. C. Gastro duodenal irritation C. Hemostasis D. D. Acute deterioration of renal function _____ 7. Effective in neuropathic pains: A. Opioids B. C. Antidepressants C. NSAIDs D. D. Anticonvulsants _____ 8. Acupuncture based on traditional Chinese medicine.

A. B. C. D.

Solid needles inserted into trigger bonds Solid needles inserted along meridians Local anesthetic injected into or near trigger bonds Electrical stimulation along trigger bonds

_____ 9. A focussed concentration used as a guide to patients to focuss away from their pain. A. Biofeedback C. Aromatherapy B. Hypnosis D. Relaxation exercises _____ 10. Which of the following NSAIDs act on the central cyclooxygenase? A. Ibuprofen C. Aspirin B. Paracetamol D. All of the above _____ 11. The gold standard to which all of the other opioids are being compared to. A. Demerol C. Nalbuphine B. Morphine D. Fentanyl _____ 12. Which of the following opioids is an agonist-antagonist. A. Demerol C. Nalbuphine B. Morphine D. Fentanyl _____ 13. Mechanism of action of ketamine. A. Inhibits adenyl cyclase in the post-synaptic nerve B. Enhances GABA receptor in the pre-synaptic nerve C. Prevents the removal of magnesium from the NMDA receptor in the post-synaptic nerve D. Prevent substance P. from occupying NKI receptor in the post synaptic cell membrane. _____ 14. Opioids receptor responsible for the respiratory depressant effect of opioids A. Mu 1 receptor C. Kappa 1 receptor B. Mu 2 receptor D. Kappa 2 receptor _____ 15. The following local anesthesic exhibit vasoconstricting effect at low and high doses. A. Lidocaine C. Ropivacaine B. Bupivacaine D. Cocaine _____ 16. One of the following is on objective sign of CNS toxicity: A.Convulsion C. Drowsiness B. Dizziness D. Lightheadedness _____ 17. Local anesthetic that belongs to amide group. A. Procaine C. Cocaine B. Lidocaine D. Tetracaine _____ 18. Local anesthetic that is converted to O-toluidine in the liver: A. Procaine C. Prilocaine B. Etidocaine D. Cocaine _____ 19. Local anesthetic with arrtiarrhythmic effect A. Tetracaine C. Lidocaine B. Bupivacaine D. Etidocaine _____ 20. Addition of vasoconstrictors to local anesthetics will: A. shorten the duration of action of local anesthetic B. Prolong the duration of action of local anesthetic C. Increase the potency of the drug D. Decrease the potency of the drug _____ 21. Local anesthetic that causes unidirectional block and re-entry type of cardiac arrhythmia. A. Bupivacaine C. Lidocaine B. Etidocaine D. Tetracain _____ 22. The median nerve originates from the: A. Lateral cord C. Both A & B B. Medial cord D. Neither A & B _____ 23. The Dorsalis Pedis Artery is used as landmark to block the: A. Superficial Peroneal nerve B. C. Deep Peroneal nerve C. Saphenous nerve D. D. Sural nerve _____ 24. The nerve blocked, when local anesthetic is injected between the palmaris longus and tendon longus and flexor carpi radialis at the wrist. A. Ulnar nerve C. Radial nerve B. Median nerve D. None of the above _____ 25. Caudal anesthesia is a form of: A. Spinal anesthesia C. Local block

B. Peripheral block D. Epidural block _____ 26. Boundaries of the anatomic Snuff Box EXCEPT: A. Extension Pollicis Longus C. Extension Pollicis Brevis B. Abduction Pollicis Longus D. Abductor Pollicis Brevis _____ 27. The nerve blocked posterior to the lateral malleolus. A. Sural nerve C. Saphenous nerve B. Tibial nerve D. Superficial peroneal nerve _____ 28. Complication of pudendal nerve block during vaginal delivery: A. Inadvertent IV injection B. C. Puncture of fetal head C. Injury to the nerve D. D. Toxicity to local anesthetic _____ 29. Delayed complication of spinal anesthesia A. Difficulty of breathing C. Hypotension B. Urinary retention D. Nausea _____ 30. The nerve found at the anatomic snuff box nerve A. Median Nerve C. Radial nerve B. Ulnar Nerve D. Digital Nerve _____ 31. Nerve blocked in Retrobulbar nerve block A. Occulomotor nerve C. Ophthalmic nerve B. Ciliary Ganglion D. Optic nerve _____ 32. Branch of intercostals nerve often spared when injection of local anesthetic is done at the anterior axillary line: A. Anterior cutaneous branch B. C. Posterior cutaneous branch C. Medial cutaneous branch D. D. Lateral cutaneous branch _____ 33. Landmark in doing superficial cervical plexus block. A. Internal jugular vein B. C. Anterior scalene muscle C. Sternocleidonastoid muscle D. D. Omohyoid muscle _____ 34. Approach in doing brachial plexus block that anesthetize the entire plexus: A. Supraclavicular Approach B. C. Interscalene Approach C. Infraclavicular Approach D. D. Axillary Approach _____ 35. Post spinal headache as a delayed complication of spinal anesthesia is due to: A. Hamatoma B. C. Muscle strain C. CSF leakage D. D. Referred pain _____ 36. T6 level of sensory blockade is at the level of : A. Nipple line C. Umbilius B. Subcostal line D. Xiphoid process _____ 37. The line joining the highest points of iliac crest (ASIS) crosses: A. L1 interspace C. L4 interspace B. L3 interspace D. L2 interspace _____ 38. In adult the spinal cord ends at the level of: A. S1 C. L1 B. S2 D. L3 _____ 39. Drug administration that allows patient to titrate analgesics according to their needs: A. Patient controlled analgesia (PCA) B. Intravenously on a per need basic C. Transdermal therapeutic system D. Oral on a time contingent basis _____ 40. A A. B. C. D.

patient can be discharged from the Postoperative care unit if he is: Drowsy but responds to pain Blood pressure is more than 30% of baseline Still in pain even if analgesics has been given Pulse oximeter reading is 97% at room air.

CASE: LA, 35 years old female is schedule for appendectomy: BP=130/80 PR=88/min anesthesia given is subarachnoid block: _____ 41. What is the ideal level of the block?

A. B.

High spinal Midspinal

C. Low spinal D. Saddle block

_____ 42. Upon doing the pinprick test, the sensory block is at the level of the nipple (T4). Give the level of motor block: A. T6 C. T2 B. T8 D. T10 _____ 43. Five minutes after assuming the supine position; the BP because 90/60 PR=50/min Give the reason for hypotension: A. Para sympathetic block B. C. Sympathetic block C. Motor block D. D. Sensory block _____ 44. What is the immediate management? Of hypotension A. Head up position B. C. oxygenation C. Increase IV fluid infusion D. D. Give vasoconstrictors _____ 45. What is the immediate management of the bradycardia? A. Ephedrine sulfate IV B. C. Intravenous fluid infusion C. Atropine sulfate D. D. Midazolam IV _____ 46. Intake of an agent by the body and the uptake of the agent by the tissues is termed. A. Circulation C. Absorption B. Elimination D. Metabolism _____ 47. Distribution of general anesthetic drugs is influenced by: A. Route of administration B. C. Bioavailability C. Regional blood flow D. D. Metabolites _____ 48. Discovered oxygen and nitrous oxide: A. John snow C. Crawford T. Long B. Joseph Priestly D. Henry Hickman _____ 49. Muscle relaxation is the result of : A. Mental block C. Motor block B. Sensory block D. Block of reflexes _____ 50. True of nondepolarizing muscle relaxant: A. Competetive inhibition with acetylcholine B. Prolonged depolarization C. Reduced sensitivity to post junctional membrane D. Action is reversed by acetylcholinesterases _____ 51. Last CNS structure depressed by general anesthesia is: A. Cortical center C. Medullary center B. Spinal cord D. Basal ganglion & cerebellum _____ 52. A 16 year old healthy patient with incarcerated inguinal hernia, for inguinal herviorhaphy the American Society of Anesthesiologists risks classification under: A. Class I C. Class III B. Class II D. Class IV _____ 53. Cardiovascular intolerance to changes in position after anesthesia is a sign of: A. Insufficient depth of anesthesia B. C. Excessive depth of anesthesia C. Sufficient depth of anesthesia D. D. All of above _____ 54. Heat and cold sensations are transmitted through this type of nerve fibers: A. C- Fibers B. C. A beta fibers C. A- Delta fibers D. D. All of the above _____ 55. Sharp cut on the skin are transmitted mostly through this type of nerve fibers: A. C-fibers C. A beta fibers B. A delta fibers D. All of the above _____ 56. Referred pain occurs due to convergence of nerve fibers in this neurons: A. Nociceptive specific neurons

B. C. D.

C. Brain stem neurons Wide dynamic range neurons D. All of the above

_____ 57. Which of the following is an excitatory nerve transmitter for the pain pathway: A. Glutamate C. Acetylcholine B. Serotonin D. Norepinepherine _____ 58. In the dorsal horn of the spinal cord, the opioid receptor are found in: A. Lamina I C. Lamina III B. Lamina II D. Lamina V _____ 59. First order neuron are found in: A. Dorsal horn B. C. Thalamus C. Ventral root ganglion D. D. Dorsal root ganglion _____ 60. Primary afferent fibers with fastest conduction velocity: A. C type fiber C. A beta fibers A delta nerve fiber D. D. fibers _____ 61. Known as carrier gas: A. Nitrous oxide B. Oxygen

C. Carbon dioxide D. Carbogen

_____ 62. True of ketamine HCL effect: A. Lower intracranial pressure B. C. Increase intraocular pressure C. Lower blood pressure D. D. Increase awareness _____ 63. Which of the following is a belladona alkaloid? A. Diphenhydramine HCL B. C. Morphine sulfate C. Atropine sulfate D. D. Nalbuphine _____ 64. An inherited abnormality resulting from exposure of patient to certain anesthesic: A. Malignant hypothermia B. C. Benigs hyperthermia C. Malignant hyperthermia D. D. Benigs hypothermia _____ 65. One of the following is a weak anesthetic but a potent analgesic. A. Nitrous oxide C. Oxygen B. Carbon dioxide D. Ethyline _____ 66. Local anesthetic with vasoconstriction effect: A. Lidocaine C. Pontocaine B. Bupivacaine D. Cocaine _____ 67. Systemic toxicity to local anesthesia is manisfested by: A. Convulsions B. C. Methemoglobinemia C. Skeletal muscle irritation D. D. Addiction _____ 68. Causes methemoglobinemia because it is degraded in the liver to otoluidine w/c causes oxidation of hemoglobine. A. Xylocaine C. Prilocaine B. Coccaine D. Bupivacaine _____ 69. Factors that affect absorption of local anesthetic. A. Site of injection C. pKA B. Protein buiding D. pCO2 ______ 70. Duration of action of local anesthetic correlates with : A. Lipid solubility C. pKA B. Protein buiding D. Degree of Ionization ____ 71. Potency of a local anesthetics correlates with. A. Lipid solubility C. PKA B. Protein buiding D. Degree of Ionization _____ 72. Onset of action of a local anesthetic correlates with. A. Lipid solubility C. pKA B. Protein buiding D. Degree of Ionization _____ 73. When asked about the pain he categorically says it is minimal. Numerical

score is. A. 1 to 3 B. 5 to 6

C. 8 to 10 D. zero

_____ 74. Tramadol and Meperidine should not be given in patients receiving. A. Adenycyclase inhibitor B. C. Lipooxygenase inhibitor C. Cyclioxygenase inhibitor D. D. Monoamineoxydase inhibitor _____ 75. Mechanism of action of tramadol. A. A weak mu receptor agonist B. Inhibits nor adrenalin receptor C. Inhibits serotonin receptor D. All of the above _____ 76. Mechanism of action of NSAIDs A. Inhibition of lipooxygenase B. Inhibition of cyclioxygenase inhibitor

C. Inhibition of phospholipase D. All of the above

_____ 77. Common complication of retrobulbar block in extremely myopic eye. A. Globe perforation C. Paralysis of retrobulbar nerve B. Retrobulbar hemorrhage D. Blindness _____ 78. The landmark for doing deep peroneal nerve block. A. Dorsalis pedis vein C. Dorsalis pedis artery B. Medial malleolus D. Lateral malleolus _____ 79. Most common complication of intercostals nerve block. A. Pneumothorax C. Convulsions B. Intravascular injectionD. Failed block _____ 80. Local anesthetic technique which uses lidocaine spray 10%. A. Field block C. Bier block B. Infiltration D. Topical _____ 82. Needle puncture during spinal anesthesia can be made safely at the interspace. A. Between L1 – L2 C. L4 – L5 B. T12 – L1 D. L5 – S1 _____ 83. Mechanism of chronic pain which is often referred to as “nociceptive pain”. A. Peripheral C. Central peripheral B. Central D. Psychophysiologic _____ 84. Management for central type pf pain: A. NSAIDs C. Aspirin B. Opioids D. Steroids _____ 85. Nerve missed in doing the axillary approach in brachial plexus block A. Axillary nerve C. Ulnar nerve B. Lateral rectoral nerve D. Musculo cutaneous nerve _____ 86. Nerve blocked posterior to the medial malleolus. A. Sural nerve C. Superficial peroneal nerve B. Tibial nerve D. Saphenous nerve _____ 87 . A. B. C. D.

The anesthetic potency of volatile liquid anesthetic is measured by: Minimum arterial concentration Minimum alveolar concentration Maximum alveolar concentration Maximum airway concentration

_____ 89. The most common cause of airway obstruction is general anesthesia is: A. Salivary secretions B. C. Presence of enlarged adenoids C. Falling back of the tongue D. D. Enlarged tonsils _____ 90. Green is the standard color code for : A. Carbon dioxide B. C. Carbogen C. Oxygen D. D. Nitrous oxide _____ 91. Elimination of general inhalation anesthetics is mainly thru: A. Kidney C. Blood B. Liver D. Lungs _____ 92. Part of the anesthesia machine that releases gas to the atmosphere. A. Vaporizer B. C. Pop-off value

C. D.

Pressure reducing value D. Flow meter

_____ 93. A patient with severe systemic disease that is a constant threat to life, will have a physical status classification. A. ASA I C. ASA III B. ASA II D. ASA IV _____ 94. Lowest intensity at which a given stimulus is perceived as painful. A. Pain tolerance C. Pain suffering B. Pain threshold D. All of the above _____ 95. One of the following is an example of acute pain. A. Arthralgia C. Headache B. Low back painD. Post-operative pain _____ 96. This gas is known as a “Laughing Gas” A. Carbon dioxide C. Nitrous oxide B. Ethylene D. Oxygen _____ 97. Type of ventilation used for apneic patient. A. Spontaneous ventilation C. Assisted ventilation B. Controlled ventilation D. Positive end expiratory pressure ventilation _____ 98. Transduction occurs in the : A. Peripheral terminals of primary afferent neurons B. Projections of spinal neurons C. Properties of diencephalic neurons D. Projections of wide dynamic range neurons _____ 99. Characteristics of acute pain: A. Generate reflexes B. C. Long duration intervals for months C. Associated with depression D. D. Recurs at intervals for months _____ 100. The nerve fiber is freely permeable to: A. CL – ions C. Na + ions B. K + ions D. Ca + ions B

2

Which of the following layers of the retina does not terminate at the optic disc margin: A. Retinal pigment epithelium B. Nerve fiber layer C. Ganglion cell layer D. Layer of rods and cones

23-24

C

3

The A. B. C. D.

A

4

The orbital space formed by the recti muscles and their intermuscular membrane with the Tenon‟s capsule: A. Central surgical space B. Subperiosteal space C. Episcleral space D. Peripheral surgical space

C

5

The condensation of fibrous tissues that thickens to form a sling (hammock) upon which the globe rests: A. Whitnall‟s ligament B. Check ligament C. Ligament of Lockwood D. Annulus of Zinn

D

6

Which of the following does not pass thru the Superior orbital fissure: A. Abducens B. Trochlear C. Oculomotor D. Optic nerve

B

7

The A. B. C. D.

A

8

The adult size of the cornea is attained at: A. 6th year of life

adult orbital measurement is attained at this age: 5th year 3rd year 9th year 20th year

35

36

39

37

extrinsic muscle that is not inserted in front of the equator: Medial rectus Superior rectus Lateral rectus Inferior rectus

40

3

B. C. D. C

10

B

20

at birth 3rd year of life 9th year of life

Which of the following is not TRUE of the Meibomian gland: A. Does not communicate with hair follicles B. Responsible for secreting the oily layer of the tear film C. Modified sweat gland D. Originates from the tarsus True of the Optic nerve, except:

46

60-61

A. B. C. D.

Consists of about 1 million axons from the ganglion cells of the retina. Emerges from the posterior aspect of the globe. Myelinated all throughout. None of the above

B

21

True of Conjunctival injection: A. Vessels most numerous at the limbus and fade toward the fornix B. Vessels constrict with a drop of 1:1,000 epinephrine C. Associate with keratitis, iridocyclitis, and angle closure glaucoma D. Cornea cloudy, pupils distorted, vision reduced

C

22

Marked enlargement of the optic disk is rarely seen, but its occurrence suggests: A. myopia B. posterior staphyloma C. both D. neither

A

33

Any A. B. C. D.

opacity in the lens is called: Cataract Leukoma Nebula None of the above

D

34

The A. B. C. D.

optic nerve is actually a collection of axons of the: cells of the inner nuclear layer cells of the outer nuclear layer photoreceptors ganglion cells

D

35

As it courses through the orbit, the optic nerve derives its blood supply from the: A. pial vessels B. arachnoid vessels C. vessels of the dura D. d. central retinal artery

B

C

A

211-212

353-354

369

36 The center of the visual field corresponds to visual perception from the following anatomic structure: A. the optic nerve B. the fovea C. the ora serrata D. none of the above 37 Mass lesions of the pituitary gland classically produce the following field pattern: A. quadrantic hemianopsia B. binasal hemianopsia C. bitemporal hemianopsia D. centrocecal scotoma 38 To enable stereoscopic vision, the visual fibers are so arranged that fibers over the nasal retina decussate while temporal fibers continue ipsilaterally to their corresponding lateral geniculate body. The crossover of fibers occur at the: A. optic chiasm B. orbit C. cannaliculus D. none of the above

353

352

295

531

66-67

B

39 Despite the seemingly horrendous appearance of the optic disc in acute papilledema, vision remains fairly good and the only deficit in the visual field is a: A. B. C. D.

cecocentral scotoma enlarged blind spot bitemporal hemianopsia central scotoma following is true of optic nerve meningiomas: occurs most frequently during the first 2 decades of life symptoms are different from that of sphenoid wing meningiomas visual loss mainly through compression or atrophy more symptomatic than gliomas

B

41

The A. B. C. D.

B

42

Ischemic optic neuritis presents with this classical field defect: A. B. C. D.

357

364

360

enlarged blind spot cecocentral scotoma altitudinal field defect bitemporal hemianopsia

A

50

True of pinguecula: A. benign degenerative tissue B. tumor of the eyelids C. more common in the young D. all of the above

214

D

51

True of symblepharon: A. chief cause is chemical burn B. keratitis occurs because of exposure C. obliterate conjunctival cul-de-sac D. all of the above

C

74

Middle coat of the eye is composed of the following, except: A. Iris B. Ciliary body C. Retina D. Choroid

D

75

Structure that regulates the amount of light reaching the visual receptors of the eye: A. Cornea B. Aqueous C. Vitreous D. Iris

B

76

The A. B. C. D.

A

90

Portion of the Tenon capsule that forma a sling upon which the globe rests: A. ligament of lackwood B. lower tendon of zinn C. whitnall‟s ligament D. check ligament

215

15-16

embryologic tertiary vitreous is known as what structure in the adult eye: Ora serrata Vitreous body Zonules None of the above

20

72

39

OPHTHALMOLOGY QUESTIONS Conjunctivitis 1. The type of conjunctivitis characterized by a localized whitish nodule with a necrotic excavated center surrounded by conjunctival injection: a. epidemic keratoconjunctivitis

b. c. d.

corneal ulcer phlyctenular conjunctivitis vernal conjunctivitis

(Espiritu etal, p. 54)

Cataract 2. The virus that can cause congenital cataract: a. adenovirus b. picornavirus c. rubella (Newell, p. 377) d. rubeola

Glaucoma 1. Carbonic anhydrase inhibitors are given as a treatment for glaucoma. Its mechanism of action is: a. to increase the aqueous humor outflow facility b. to decrease aqueous humor production (Espiritu, p. 101) c. to decrease production of aqueous and enhance its exit d. to increase aqueous humor production Cornea 2. While cutting grass in his garden, a 50-year old male felt a small piece enter his left eye. He experienced foreign body sensation and eye redness. No consult was done and no medications were instilled. A few days later, he noted a fluffy, grayish white elevation on the center of his eye. Blurring of vision was also noted. Which of the following medications is contraindicated? a. atropine sulfate b. 5% natamycin c. corticosteroids (Newell, p. 249) d. amphotericin B 3.

Retina 4.

Softening of the cornea associated with malnourishment: a. keratoconus b. keratomalacia (Espiritu etal, p. 60) c. Mooren‟s ulcer d. Staphyloma An ophthalmic emergency characterized by sudden blurring or complete loss of vision with a characteristic cherryred spot on funduscopy: a. central retinal vein occlusion b. branched retinal artery occlusion c. branched retinal vein occlusion d. central retinal artery occlusion (Espiritu, p. 88)

5.

A 45-year old female suddenly experienced blurring of vision, with a sensation of a curtain in a part of the field of vision. The primary consideration in this patient is: a. diabetic retinopathy b. retinal detachment (Espiritu, p. 92) c. central serous chorio-retinopathy d. hypertensive retinopathy

6.

A 2-year-old boy was brought in for consultation because the mother noted a “white pupil”. Which of the following is NOT a differential diagnosis for this case? a. congenital glaucoma (Newell, p. 326) b. retrolental fibroplasias c. persistent hyperplastic primary vitreous d. toxocariasis

7.

Which of the following is responsible for scotopic vision or dim illumination: a. rods only (Espiritu, p. 86) b. cones only c. both A & B d. none of the above Systemic Ophthalmology 8. The ocular lesion in congenital toxoplasmosis is characterized as:

a. b. c. d.

corneal dendritic ulcer which stains with fluorescein follicular conjunctivitis, optic neuritisand non-granulomatous uveitis single choroidoretinal scar prominently in the posterior fundus (Espiritu, p.126) big necrotic lesion in the retina and choroids surrounded by smaller lesions

Lacrimal apparatus 9. A 45-year-old male presents with a painless mass on the upper outer portion of his upper eyelid. There was moderate proptosis. Which is true of his condition? a. It is the second most common epithelial tumor of the lacrimal gland b. It has the worst prognosis c. It contains mesenchymal elements and double-layered tubular epithelial units (Newell, p. 263) d. It is composed of aggregates of small undifferentiated neoplastic cells separated by small and large cystoid spaces with mucin Eyelids 10. A chronic inflammatory lipogranuloma of a meibomian gland, characterized by a gradual painless swelling of the gland without other external signs of inflammation: a. external hordeolum b. chalazion (Newell, p. 204) c. meibomianitis d. internal hordeolum 11. The palpebral a. 8-12 b. 9-12 c. 8-10 d. 8-10

fissure among Filipinos measures: mm in height and 25-29 mm in length mm in height and 26-29 mm in length mm in height and 25-29 mm in length mm in height and 26-29 mm in length ( Espiritu, p. 9)

Neuro-ophthalmology 12. Passive swelling of the optic nerve occurring secondary to increased pressure in the subarachnoid space of the meningeal coverings of the brain and optic nerves: a. papilledema (Newell, p. 357) b. pseudo-papilledema c. drusen d. optic neuritis

73

View more...

Comments

Copyright ©2017 KUPDF Inc.
SUPPORT KUPDF